Old Q&A – Gastroenterology

 

A 65-year-old Asian American man comes to the clinic for a follow-up appointment for symptoms of dysphagia. He has had difficulty swallowing solid food off and on for the past year. He has no difficulty swallowing liquids or pills. He has no significant medical problems and his only medication is an occasional aspirin for arthritis pain in his knees. He denies cigarette smoking, but he does drink 1-2 glasses of wine each week. An outpatient esophagram was performed 3 days ago and the x-ray shown is one of the films obtained during the study. Based on the findings of the esophagram, this patient is at increased risk for developing

  A. Barrett’s esophagus
  B. esophageal cancer
  C. gastric ulcers
  D. gastroesophageal reflux
  E. Zencker’s diverticulum
Explanation:

The correct answer is E. The esophagram demonstrates an indentation on the posterior cervical esophagus, which is due to a hypertrophied cricopharyngeus muscle. A Zencker’s diverticulum is a pharyngoesophageal pulsion diverticulum that occurs at the point of transition between the oblique fibers of the thyropharyngeus muscle and the horizontal fibers of the cricopharyngeus muscle. A hypertrophied cricopharyngeus muscle is thought to predispose the development of the diverticulum at this transition point. This can be an incidental finding or in some patients, as in this case, it can cause symptoms of dysphagia.

Barrett’s esophagus (choice A) is a complication of gastroesophageal reflux disease. There is metaplasia of the normal cells in the esophagus to what is called “specialized columnar epithelium”. Chronic damage from the acidic gastric contents is believed to promote the replacement of the normal esophageal epithelium with the metaplastic columnar cells. This is a predisposing condition for adenocarcinoma of the esophagus. There is no association with hypertrophy of the cricopharyngeus muscle.

There is no relationship between a hypertrophied cricopharyngeus muscle and esophageal cancer (choice B).

There is no relationship between a hypertrophied cricopharyngeus muscle and gastric ulcers (choice C).

There is no relationship between a hypertrophied cricopharyngeus muscle and gastroesophageal reflux (choice D). Reflux can occur due to a hiatal hernia or a dysfunctional lower esophageal sphincter.

http://www.usmlestep.com

http://health.groups.yahoo.com/group/usmlestep3

Visit us to get more STEP 3 Materials

 

  A 55-year-old banker comes to the office for a routine initial visit. He says that he has no significant past medical or surgical history. He takes no medications except for a daily multivitamin. His family history reveals that his mother died of breast cancer and his father committed suicide when he was 5 years old. On further questioning, he reports that he has been working 10 to 12 hours a day and is feeling very stressed at work. He has been drinking 2 to 3 beers a night to help relieve the stress and help him get some sleep. He vehemently denies ever experimenting with intravenous drugs, but currently uses marijuana about once a month. He has had 3 different sexual partners over the past month and does not routinely use condoms. Physical examination reveals a nodular liver edge of 9-cm in diameter and a tender abdomen in the right upper quadrant but no rebound or guarding. His Murphy’s sign is negative. His liver function panel shows:

The factor in this patient’s history most closely correlated with his condition is

  A. alcohol intake
  B. drug use
  C. family history of cancer
  D. His history correlates with biliary tract disease.
  E. unsafe sexual practices
Explanation:

The correct answer is E. This patient has multiple different reasons to have liver damage and cirrhosis. The liver function panel indicates that this patient probably has cirrhosis, as indicated by synthetic liver disease (coagulopathy and decreased albumin). His transaminases are more consistent with a viral etiology than alcoholic hepatitis (ALT>AST), probably viral hepatitis from a sexual encounter.

Alcoholic hepatitis (choice A) commonly causes liver disease characterized with an elevated AST > AST. Additionally, cirrhosis secondary to alcoholic hepatitis routinely causes an enlarged liver edge rather than a normal or small-sized liver.

The most common cause of viral hepatitis/cirrhosis in the United States is hepatitis C as a result of intravenous drug use (choice B). However, this patient denies ever using intravenous drugs and given his candor during the interview, there is no reason to suspect that he is lying. Marijuana has not been linked to hepatitis.

The patient’s family history of cancer (choice C) doesn’t correlate to an increased risk of liver disease.

This patient has a negative Murphy’s sign and a normal alkaline phosphatase. It is incorrect to say that nothing in his history suggests biliary disease (choice D), because his unsafe sexual practices increase his risk of hepatitis.

 

 

  A 51-year-old man is post-procedure day number 2 from an upper endoscopy and banding for bleeding esophageal varices. The patient has a 7-year history of chronic active hepatitis and over the past few years has developed stigmata associated with cirrhosis and worsening portal hypertension. Three days ago, he presented to the emergency department with bright red blood per mouth and rectum and a nasogastric tube evacuated bright red blood and coffee grounds from the patient’s stomach. He was admitted to the hospital, transfused with 2 units of red blood cells and underwent an endoscopy. On preparation for the patient’s discharge, you have a long discussion with your patient about the course of events. In counseling this patient on his future risks and course of therapy, you should advise him that:

  A. The risk of rebleeding is between 50% and 80% and medical therapy is indicated
  B. The risk of rebleeding is between 50% and 80% and surgical therapy is indicated
  C. The risk of rebleeding is between 50% and 80%, but no therapy is indicated given the nature of the problem
  D. There is no concern for rebleeding
  E. There is no concern for rebleeding, he is an imminent candidate for transplant
Explanation:

The correct answer is A. There is overwhelming data to support the treatment strategy of some sort of medical therapy (beta blockade, sclerotherapy, banding) for all patients with an episode of variceal bleeding. The risk of rebleeding is high and approaches 80% in most series at 1-year. There is excellent data showing the efficacy of portal-systemic surgical shunts (side-to-side caval, end-to-side caval, distal splenorenal, TIPS) in secondary prevention. Medical therapy is indicated in all patients and all patients should be offered the option of a surgical shunt. The largest drawback to these procedures is their resultant incidence of encephalopathy.

The risk or rebleeding is between 50% and 80% and surgical therapy is indicated (choice B) is incorrect because there are nonsurgical options available. These options include beta blockade, sclerotherapy, and banding.

It is incorrect to say that the risk of rebleeding is between 50% and 80% but no therapy is indicated given the nature of the problem (choice C). As stated above, therapy options include beta blockade, sclerotherapy, banding, and hopefully a liver transplant.

There is a 50% and 80% chance of rebleeding, therefore, there is no concern for rebleeding (choice D) is incorrect.

Liver transplant (choice E) is of course curative and its 5-year survival rate of approximately 70% is superior to cirrhotics with Child class C disease who are treated by other modalities. This answer is incorrect because it starts with “there is no concern for rebleeding,” which is incorrect.

 

  A 58-year-old woman comes to the emergency department complaining of crampy left upper quadrant pain that is exacerbated by fatty foods. She has a history of diabetes, hyperlipidemia, and gallstones and her medications include glyburide, simvastatin, and aspirin. She denies any alcohol or drug use. She is morbidly obese and her temperature is 37.9 C (100.2 F), blood pressure is 102/87 mm Hg, pulse is 105/min, and respirations are 23/min. On examination, her lungs are clear to auscultation bilaterally. Her cardiac sounds are muffled, although her cardiac rhythm is regular. No murmurs are audible. She has definite left upper quadrant tenderness to palpation, without rebound or guarding. Rectal examination shows guaiac-negative brown stool. Her amylase and lipase levels are elevated. The most appropriate next step is to order a

  A. an abdominal ultrasound
  B. a chest radiograph
  C. an electrocardiogram
  D. a HIDA scan
  E. an upper endoscopy
Explanation:

The correct answer is A. Given that this patient has pancreatitis, it is reasonable to suspect gallstones as its etiology given the available epidemiologic factors (sex, age, obesity). If gallstones are present, an ERCP and gallstone removal can be considered.

A chest radiograph (choice B) is not indicated at this juncture given that her lung exam is benign and there is no evidence of a pulmonary pathology.

An electrocardiogram (choice C) is not indicated at this juncture since cardiac pathology is not evident on history or exam and the patient appears to be in sinus rhythm.

A HIDA scan (choice D), sometimes used for the diagnosis of cholecystitis, is not used in the diagnosis of pancreatitis. An upper endoscopy (choice E) is not indicated in the setting of pancreatitis without confounding pathologies such as variceal bleeding or gastric ulceration.

 

 

  A 55-year-old woman comes to the emergency department because of abdominal pain. She had just finished eating a steak dinner with her family when she suddenly experienced sharp, crampy pain in the upper right and middle of her abdomen. The pain has lasted for the past 3 hours and she is starting to feel nauseous. On physical examination, she is obese and in obvious discomfort. Her temperature is 38.8 C (101.8 F), blood pressure is 140/87 mm Hg, pulse is 90/min, and respirations are 16/min. Abdominal examination is significant for focal tenderness and guarding in her right upper quadrant. She is particularly tender when you palpate her right upper quadrant as she takes in a deep breath. The most appropriate next step in the evaluation of her abdominal pain is

  A. an abdominal x-ray
  B. a CT of the abdomen
  C. an endoscopic retrograde cholangiopancreatography (ERCP)
  D. serum liver function tests including bilirubin
  E. an ultrasonography of the abdomen
Explanation:

The correct answer is E. Crampy pain in the right upper quadrant or epigastrium is classic for biliary colic. The pain is thought to be due to obstruction of the cystic duct by gallstones and often follows a fatty meal. Acute cholecystitis is a concern when biliary colic is accompanied by a fever, leukocytosis, nausea, and vomiting. Right upper quadrant pain precipitated by deep inspiration during palpation is known as Murphy’s sign and is highly suggestive of acute cholecystitis. The presence of gallstones in the clinical setting of right upper quadrant pain and fever is sufficient to make the diagnosis of acute cholecystitis. Ultrasonography of the abdomen is the diagnostic test of choice to evaluate for gallstones. Ultrasonography is highly sensitive and specific for gallstones, and can also demonstrate inflammation of the gallbladder. During ultrasonography, a sonographic Murphy’s sign can also be tested for by using the transducer to press over the region of the gallbladder. The presence of gallstones and a sonographic Murphy’s sign are highly suspicious for acute cholecystitis.

An abdominal x-ray (choice A) is not a good study to evaluate for gallstones. Most gallstones are composed of cholesterol and are radiolucent on abdominal x-rays. Only 10-15% of gallstones contain enough calcium to appear radio opaque on x-rays.

A CT of the abdomen (choice B) is not the first or the best test in the evaluation of gallstones or acute cholecystitis. Most gallstones are composed of cholesterol and may not be clearly distinguishable from adjacent bile in the gallbladder on a CT scan. A CT scan provides information about inflammation around the gallbladder and biliary ductal dilatation, but it is not a sensitive or specific diagnostic study for gallstones.

An endoscopic retrograde cholangiopancreatography or ERCP (choice C) is performed by gastroenterologists to evaluate the biliary ductal system. It is the procedure of choice when a common bile duct stone or choledocholithiasis is suspected. When a patient has gallstones and common bile duct dilatation on ultrasonography of the abdomen, ERCP is useful in the evaluation of the biliary system as well as for clearing the obstructing stone. ERCP is not indicated in cases of simple biliary colic or acute cholecystitis without signs of common bile duct obstruction.

Serum liver function tests including bilirubin (choice D) can provide useful information about biliary obstruction. Elevated bilirubin and frank jaundice are clinical indicators of biliary obstruction, most commonly in the common bile duct. These serum laboratory tests do not help in the diagnosis of gallstones or acute cholecystitis.

           

A 78-year-old woman comes to the geriatric clinic for a follow-up appointment. She was seen 3 weeks ago in the clinic for a routine appointment and was found to have a hematocrit of 28%. A rectal examination was positive for heme in the stool. Her only complaint is a long history of constipation. She has multiple medical problems including diabetes, hypertension, osteoarthritis, and a history of a myocardial infarction many years ago. To further evaluate her anemia, additional laboratory testing was initiated at that time. Since her last appointment, she had an outpatient barium enema and is now returning for the results of all her tests. An x-ray of the recto-sigmoid colon from the barium enema examination is shown. Serum laboratory tests are as follows:

At this time, the most appropriate next step is to

  A. admit the patient to the hospital for further evaluation
  B. do a colonoscopy
  C. encourage the patient to eat a high fiber diet
  D. order serum carcinoembryonic antigen (CEA-125)
  E. prescribe iron supplements
Explanation:

The correct answer is B. A colonoscopy is the most appropriate next step in the management of this patient. Her laboratory tests reveal a microcytic anemia with a low ferritin, which is consistent with an iron deficiency anemia. This is a common presentation of colon cancer. The barium enema demonstrates an irregular lesion in the sigmoid colon that is highly suspicious for an adenocarcinoma of the colon. A colonoscopy can be performed to evaluate the entire length of the colon for polyps and masses. It can better characterize the extent of the lesion detected on the barium enema, as well as allow for a biopsy of the lesion for definitive tissue diagnosis.

Admitting the patient to the hospital for further evaluation (choice A) is not necessary in this situation. Patients with colon cancer who present with symptoms of bowel obstruction or perforation will need to be hospitalized to treat their acute disease. This patient is presenting with more indolent symptoms of colon cancer such as iron deficiency anemia and chronic constipation. She can be safely evaluated as an outpatient.

Encouraging the patient to eat a high fiber diet (choice C) is not appropriate management. She has a lesion on the barium enema that is highly suspicious for malignancy and requires further evaluation with colonoscopy. A high fiber diet is recommended for patients with constipation and for the prevention of diverticulosis.

Ordering a serum carcinoembryonic antigen (CEA-125) (choice D) is not the best step in the management of this patient. CEA-125 is a serum glycoprotein frequently used in the management of patients once they are diagnosed with colon cancer. CEA-125 is not a useful screening test for colorectal cancer due to the large numbers of false-positive and false-negative reports. CEA-125 testing in postoperative patients should be restricted to patients who will be candidates for resection, in cases of liver or lung metastases. Routine use of CEA-125 alone for monitoring responses to treatment is not recommended.

Prescribing iron supplements (choice E) is not the appropriate management of this patient. Her iron deficiency anemia is a secondary sign of her primary diagnosis of colon cancer. The lesion on the barium enema is highly suspicious for malignancy and needs further evaluation with colonoscopy. Iron supplementation is good adjuvant care for this patient, but it does not address the more immediate need to make a diagnosis.

  You are seeing a 41-year-old man with alcoholic cirrhosis in your office for a follow-up visit after a recent upper endoscopy showed significant lower esophageal varices. His current medications include a multivitamin, folate, and thiamine. While he strongly denies any continued alcohol use, you are suspicious that he is still drinking. His blood pressure is 100/63 mmHg, pulse is 98/min, and respirations are 21/min. Physical examination shows a slightly protuberant abdomen. Given his varices, you are concerned about an upper gastrointestinal bleed, especially in the setting of continued alcohol use. Given this concern, the most appropriate pharmacotherapy to add to his treatment regimen is

  A. aspirin
  B. atorvastatin
  C. isosorbide mononitrate
  D. nadolol
  E. warfarin
Explanation:

The correct answer is D. Non-selective beta antagonists such as nadolol have been shown to decrease the risk of an initial variceal bleed (through a reduction in splanchnic blood flow) in someone with esophageal varices (primary prevention).

Aspirin (choice A), an antiplatelet agent, has no role in the primary prevention of variceal bleeding.

Atorvastatin (choice B), an HMG Co-A Reductase antagonist used in the management of hyperlipidemia, has no role in the primary prevention of variceal bleeding.

Nitrates such as isosorbide mononitrate (choice C) should not be started as monotherapy for the primary prevention of variceal bleeding in cirrhotics since it has been associated with increased mortality when used alone. However, it can be used in combination with beta antagonists.

Warfarin (choice E), an oral anticoagulant against vitamin K dependent clotting factors, has no role in the primary prevention of variceal bleeding.

 

  A 34-year-old intravenous drug abuser who is HIV positive is admitted to the hospital because of gastrointestinal bleeding. He was admitted to the hospital 2 months ago for HIV treatment. At the time of discharge, he was in good health, able to tolerate regular diet, and take minimal medications. He went back to work and was feeling well. Two days before presenting to the hospital, he developed nonspecific abdominal discomfort, which he attributed to food poisoning and treated himself with lots of hydration. The abdominal discomfort persisted and he noticed bleeding per rectum, the night before coming to the hospital. The next morning, he noticed more blood per rectum, and alarmed by that, decided to come to the hospital. His temperature is 37. C (99.1 F), blood pressure is 110/70 mm Hg, and pulse is 96/min. His hematocrit is 28% compared with 34% on discharge a couple of months earlier. There are no signs of hemodynamic instability. Blood is sent for cross match and stool is sent for ova and parasites. A nasogastric tube is inserted and returns clear fluid. The next step in the investigation of this patient’s gastrointestinal bleeding is a(n)

  A. barium enema
  B. colonoscopy
  C. CT scan of the abdomen and pelvis
  D. small bowel series
  E. upper gastrointestinal endoscopy
Explanation:

The correct answer is B. A colonoscopy is the initial investigation of choice in gastrointestinal (GI) bleeding in HIV-positive patients. Gastrointestinal bleeding is an unusual occurrence in HIV infected individuals, but when it does occur, it is usually related to a complication of an HIV infection. Lower GI bleeding is twice as common as upper GI bleeding. Upper GI bleeding, when it occurs, is related to Kaposi’s sarcoma or lymphoma 50% of the time. CMV ulcers do occur in the upper GI tract, but more frequently in lower GI tract. Lower GI tract bleeding is usually caused by localized colitis of infectious origin from Cytomegalovirus, herpes simplex, or bacteria. In a stable patient, colonoscopy is the procedure of choice for localizing the bleeding and obtaining biopsies to look for specific infections and antibiotic sensitivities.

A barium enema (choice A) is not as useful as the first investigation, unless a colonic carcinoma is suspected.

A CT san of the abdomen and pelvis (choice C) may show thickening of the colon with the infiltration of fat in the surrounding mesentery, but may not be diagnostic.

A small bowel series (choice D) is rarely indicated as the initial investigation of choice in investigating gastrointestinal bleeding. If upper gastrointestinal endoscopy and colonoscopy do not reveal any lesions and the patient continues to bleed, then one should look for small bowel sources by means of small bowel series or enteroscopy.

Since upper gastrointestinal tract bleeding is less common than lower gastrointestinal tract bleeding, a upper gastrointestinal endoscopy (choice E) is not indicated in this patient. Also, a nasogastric tube return does not show blood in this patient. Although it is difficult to rule out upper gastrointestinal bleeding, there is no bile in the nasogastric tube, so a colonoscopy should still be the first investigation of choice.

 

  A 3-year-old boy is brought to the emergency department by his parents because of a 24-hour history of intermittent, generalized abdominal pain. The parents tell you that he complains of the pain for 10-minute episodes and during these times he refuses to walk, but then he spontaneously returns to his normal activities. This occurred 8-9 times yesterday. Today the symptoms occurred more frequently and were associated with 3 episodes of non-bloody, non-billous emesis so the parents brought him into the hospital. There is no history of fever, constipation, or soiling. On examination the patient appears tired and has mild diffuse abdominal pain. He has guaiac-positive stool. His pulse is 125/min. The study most likely to provide a diagnosis is

  A. an abdominal x-ray
  B. a barium enema
  C. a CBC with differential
  D. a CT scan of the abdomen
  E. a lumbar puncture
Explanation:

The correct answer is B. This patient presents with a very common complaint in the pediatric population, abdominal pain. The key to this case is the quality and frequency of this abdominal pain. The pain was described as being diffuse and intermittent with periods of resolution of the symptoms. This type of pain pattern, along with emesis, the lethargy seen in the emergency department, and the guaiac-positive stools should raise red flags for the diagnosis of intussusception. In intussusception a segment of bowel (most commonly the distal ileum into the cecum) telescopes into an adjacent segment causing obstruction. This obstruction tends to resolve and recur causing the intermittent abdominal pain. The barium enema is diagnostic and in many cases a curative procedure as well and is therefore the study of choice in this case.

In a case of diffuse abdominal pain an abdominal x-ray (choice A) could show a colon full of stool and aid in a diagnosis of constipation causing the pain. That history was not seen in this case. In intussusception an abdominal x-ray might show a paucity of air in the area of the intussusception but it will most likely be inconclusive.

In intussusception, a CBC (choice C) might show us a leukocytosis, which could also be present in an infectious cause of the abdominal pain or in appendicitis, and therefore would not assist in providing a definitive diagnosis.

An abdominal CT scan (choice D) would be indicated if there was a higher index of suspicion for appendicitis, but it is not indicated as a study for suspected intussusception.

If this patient were febrile along with the lethargy, a lumbar puncture (choice E) might be warranted to rule out meningitis as a cause for the lethargy. In this case with the history as given, the spinal tap is not indicated.

 

 

A 58-year-old alcoholic with hepatitis C cirrhosis is admitted to the hospital for management of his ascites. He has been managed as an outpatient with diuretics and oral lactulose, but over the past few weeks, he reports increasing abdominal girth, weight gain and lower extremity edema. He has been noncompliant with his low-sodium diet. His medications include furosemide, spironolactone, lactulose, ciprofloxacin, and thiamine. On physical examination, he appears grossly edematous and appropriately responsive. His lungs are clear and his heart is without extra sounds or murmurs. His abdomen is tense with a fluid wave and shifting dullness on percussion. He has numerous non-blanching telangiectasias on his torso and abdomen. His testes are small for his age and there is no asterixis. Admission laboratory studies show:

Sodium 121 mEq/L
Potassium 4.3 mEq/L
Bicarbonate 29 mEq/L
BUN 38 mg/dL
Creatinine 1.5 mg/dL

Urinalysis shows some granular casts and a urinary sodium concentration of <10 mmol/L. The most appropriate therapy is to

  A. administer hypertonic saline
  B. administer sodium chloride tablets
  C. increase the dose of furosemide
  D. increase the dose of spironolactone
  E. salt restrict
Explanation:

The correct answer is E. True hyponatremia is always hypotonic. There are then three types of hyponatremia: hypovolemic, euvolemic, or hypervolemic. The therapy for the hyponatremia depends on both the urinary concentration of sodium as well as the volume status for each patient. Once the patient’s total body volume status has been estimated, urinary sodium usually allows focusing of the differential diagnosis. For this patient, he clearly has total body volume overload. He has cirrhosis and portal hypertension. The standard therapy for these patients is salt restriction and management of volume status with diuretics. This patient, at the core of his therapy requires salt restriction.

Although some forms of hyponatremia respond to administering hypertonic saline (choice A), knowing the pathophysiology of this patient’s hyponatremia, it can be clearly seen that this intervention will only aggravate this patient’s condition.

Giving the patient sodium chloride tablets (choice B) will clearly not be beneficial to this patient given the above explanations.

Increasing the dose of furosemide (choice C) or the dose of spironolactone (choice D) will certainly serve to diurese the patient, but in the presence of a salt load, the kidneys will respond by avid retention of salt and water and thus aggravate the patient’s condition.

 

  A 91-year-old woman with hypertension comes to the clinic complaining of constipation for the past 2 months. She had a hysterectomy 10 years ago and surgery for pelvic floor prolapse 6 months ago. There is hard stool in the vault on rectal examination. The stool is not grossly bloody but is heme positive. Laboratory tests reveal a hematocrit of 29% with a reticulocyte distribution width (RDW) of 33% and a carcinoembryonic antigen (CEA) of 18 ng/ml. The first test necessary to further evaluate this patient is a(n)

  A. colonoscopy
  B. colposcopy
  C. reticulocyte count
  D. ultrasound of the right upper quadrant
  E. Westergren sedimentation rate (ESR)
Explanation:

The correct answer is A. Heme positive stools and signs of colonic obstruction are common in left sided colon cancer. The elevated CEA supports the diagnosis of colon cancer. The next step is a colonoscopy to biopsy the lesion. A barium enema may also demonstrate the mass but will not allow for tissue diagnosis.

Colposcopy (choice B) utilizes a microscope to better visualize the cervix and take biopsies in women with a positive Pap smear. There is no indication for this procedure in this patient.

This patient most likely has anemia secondary to gastrointestinal blood loss as demonstrated by the elevated RDW. If a colon mass is found, further work up for this mild anemia (choice C) is not warranted.

Right upper quadrant ultrasound (choice D) is not used to stage colon cancer. A CT scan of the abdomen can be used to evaluate the local stage of the disease as well as assess for liver metastasis. Liver function tests should precede CT. Statistically, colon cancer is much more likely than a primary hepatocellular carcinoma or cholangiocarcinoma given the symptoms of constipation with heme positive stools and an elevated CEA. Colonoscopy should be performed first given the symptoms of constipation and the high likelihood of colon cancer.

ESR (choice E) is a nonspecific marker of inflammation and may be elevated in many situations, including cancer. It is of little clinical value in this patient.

 

A 35-year-old man comes to the office because of “heartburn” for 3 months. He tells you that he has a “burning sensation” in the chest that begins in the “upper stomach and travels up to the neck.” The symptoms worsen when he lies down in bed. He has no chronic medical conditions and takes no medications. He typically drinks 2-3 cups a coffee a day, has a glass of wine after dinner, and has a piece of chocolate-covered peppermint candy before bedtime. Physical examination, an electrocardiogram, a complete blood count and metabolic profile, and serologic testing for H. pylori are unremarkable. You recommend that he elevate the head of bed, avoid eating before bed, and avoid all alcohol, tobacco, chocolate, and caffeine, and schedule a follow-up visit. He comes back to the office after 2 months and says that his symptoms are unchanged. At this visit, his temperature is 37.0 C (98.6 F), blood pressure is 120/80 mm Hg, pulse is 65/min, and respirations are 14/min. Physical examination is unchanged. The most appropriate next step is to

  A. order ambulatory esophageal pH testing
  B. order an upper gastrointestinal barium radiograph
  C. prescribe famotidine
  D. schedule an upper endoscopy
  E. schedule esophageal manometry
Explanation:

The correct answer is C. This patient most likely has gastroesophageal reflux disease (GERD). Reflux disease is usually worse at night because the recumbent position allows gastric acid contents to go up into the esophagus. Since all of the tests ordered during the first visit were normal, you were correct in recommending non-pharmacologic therapy, such as elevation of the head of bed, avoiding eating before bed, and avoiding alcohol, tobacco, chocolate, and caffeine. Alcohol, tobacco, chocolate, and caffeine all lower the lower esophageal sphincter pressure leading to gastric reflux. If these measures are ineffective, pharmacologic therapy with H2 blockers such as cimetidine, famotidine, or ranitidine is indicated. For more severe symptoms, a proton pump inhibitor, such as omeprazole or lansoprazole, is indicated.

Ambulatory esophageal pH testing (choice A) is usually reserved for patients who fail nonpharmacologic and pharmacologic management.

An upper gastrointestinal barium radiograph (choice B) is useful in detecting esophageal rings or strictures, which typically present with dysphagia. This patient complains of heartburn, not dysphagia.

An upper endoscopy (choice D) is usually indicated only after the failure of nonpharmacologic and pharmacologic management for GERD and when a patient has GERD for >5 years, an upper endoscopy is recommended to screen for Barrett’s metaplasia. It is not indicated at this time.

Esophageal manometry (choice E) is typically reserved for cases of GERD when surgical therapy is being considered.

A 65-year-old woman is admitted to the hospital with severe ascites and fever. She has a 2-year history of portal hypertension secondary to hepatitis C-induced cirrhosis. The patient was placed on the liver transplant waiting list 3 months ago. Four months prior to admission she suffered an upper gastrointestinal bleed secondary to esophageal varices, which was subsequently banded via endoscopy. Two days ago, the patient developed abdominal pain, increasing abdominal girth, and fever. She was admitted to the hospital with the diagnosis of spontaneous bacterial peritonitis. The appropriate therapy is initiated and over the course of the next 4 days the patient appeared to be responding well. On the day of discharge you begin to plan her outpatient management and follow-up care. To prevent further disability from her current acute condition, you should prescribe

  A. hydrochlorothiazide
  B. lactulose
  C. levofloxacin
  D. oral protein supplements
  E. propranolol
Explanation:

The correct answer is C. The 1-year risk of recurrent infection in patients surviving spontaneous bacterial peritonitis (SBP) is as high at 70%. Prophylaxis has been tested in this group and shown to be efficacious. The rate of recurrence is less with therapy, although there is no survival benefit. The agents tested have mostly been fluoroquinolone antibiotics. Levofloxacin is now the preferred agent in that class.

Hydrochlorothiazide (choice A) is a diuretic useful in controlling ascites. It is, however, not a more important therapeutic intervention than SBP prophylaxis.

Oral lactulose (choice B) is indicated when the patient begins to suffer from hepatic encephalopathy.

Oral protein supplements (choice D) are never indicated despite the clear nutritional and total protein deficiency in these patients. The reasoning is that the metabolized nitrogen products cannot be effectively cleared and often precipitate episodes of hepatic encephalopathy.

Propranolol (choice E) is a non-selective beta-blocker used to prevent variceal bleeding. This patient has no documented varices at this time and even if they were present, the efficacy of propranolol for preventing bleeds is questionable.

 

 

A 61-year-old woman comes to the emergency department because she is “lightheaded and dizzy” after having 2 bowel movements over the past hour that consisted of bright red blood and no stool. She denies any abdominal pain or nausea, but does recall having crampy abdominal discomfort after eating over the last several days. She tells you that she has a history of “benign polyps” that are resected endoscopically every other year in her gastroenterologist’s office. Her last colonoscopy was 6 months ago and 3 hyperplastic polyps were removed. Her mother and father both passed away from complications due to colon cancer. Her temperature is 37.0 C (98.6 F), blood pressure is 100/70 mm Hg, and her pulse is 110/min. Her abdomen is non-tender and soft. There is no guarding or rebound tenderness present. There is fresh red blood in the rectum, but there are no palpable masses. Intravenous fluids are started. The most appropriate next step in management is to

  A. order a barium enema
  B. order a CT scan of the abdomen
  C. order a nuclear bleeding scan of the colon
  D. perform flexible sigmoidoscopy
  E. perform upper gastrointestinal endoscopy
Explanation:

The correct answer is D. This patient has acute lower gastrointestinal bleeding and after stabilization with intravenous fluids, requires visualization of the colon. This should first be done with a flexible sigmoidoscope, which will allow you to see the ano-rectum and determine if a lesion in this area is the bleeding source. It will also allow you to see the sigmoid colon, which is the most common site for a diverticular bleed, (one of the most common causes of a lower GI bleed).

A barium enema (choice A) should not be done in this actively bleeding patient at this time. It is often useful in patients with diverticulitis, after the acute attack subsides.

A CT scan of the abdomen (choice B) is the diagnostic study to use if this patient presented with left-sided abdominal pain, nausea, vomiting, fever, and diarrhea, the symptoms of diverticulitis, not diverticulosis.

A nuclear bleeding scan of the colon (choice C) may be helpful in identifying the exact site (but not the etiology) of the bleed; however it is often done after a flexible sigmoidoscopy or colonoscopy.

An upper endoscopy (choice E) is unnecessary at this time because it seems as if this patient has a lower GI bleed, not an upper GI bleed. A nasogastric tube can be placed and if blood or coffee-ground material is found, then the source is likely to be in the upper GI tract. However, in this case, bright red blood is seen in the rectum (and yes there is a possibility that it comes from the upper GI tract), however it seems like this “painless” bleed is lower GI in nature and requires a flexible sigmoidoscopy at this time.

 

 

A 78-year-old nursing home resident is admitted to the hospital because of increasing left-sided abdominal pain for the past 48 hours. She has had several episodes of bloody diarrhea according to the nursing attendant at the nursing home. There was no associated fever or nausea or vomiting. On admission, her temperature is 37.3 C (99.1 F), blood pressure is 90/64 mm Hg, and pulse is 100/min. Her abdomen is soft and mildly distended without masses or organomegaly. There is moderate tenderness to palpation in the left lower quadrant, but no associated peritoneal signs. Rectal examination reveals guaiac-positive stool and no masses. A flexible sigmoidoscopic examination reveals patchy, depigmented mucosa. The most appropriate initial management of this patient is

  A. angiographic embolization of the inferior mesenteric artery
  B. intravenous fluid and bowel rest
  C. mesenteric angiogram
  D. sigmoid resection and colostomy
  E. subtotal colectomy
Explanation:

The correct answer is B. Abdominal pain in an elderly patient associated with bloody diarrhea and hypotension should arouse the suspicion of ischemic bowel. In this patient, ischemic bowel is precipitated by dehydration and hypotension. A classical appearance on the flexible sigmoidoscopy of green mucosa and isolated depigmented patches are suggestive of ischemic colitis. These patients should initially be adequately hydrated and put on bowel rest. Mucosal ischemia sometimes will improve with these measures and further therapeutic measures may not be necessary.

Mesenteric angiogram and embolization (choice A) is not essential in this patient. The bloody diarrhea is from mucosal slough injury from the ischemia and will not be prevented by embolization.

An angiogram (choice C) is not necessary in the management of ischemic mucosal colitis. Ischemic colitis is diagnosed by colonoscopy. Patchy depigmented areas confirm mucosal ischemia, which is managed by intravenous fluids and bowel rest. Transmural ischemia is confirmed by green sloughing mucosa. Resection of the colon is determined by the extent of ischemia. A mesenteric angiogram may be useful in diagnosing mesenteric occlusion, but is not essential, as it is invasive, and diagnosis and guidelines for resection can be obtained by colonoscopy.

A sigmoid resection and colostomy are indicated (choice D) provided that ischemic colitis is not controlled and the patient is developing systemic signs of sepsis, which she does not currently seem to have.

Isolated pigmented ischemic colitis in the sigmoid colon (choice E) would not require a subtotal abdominal colectomy in an elderly patient.

 

 

A 50-year-old man with Crohn’s disease comes to the clinic for a routine follow-up appointment. He was diagnosed with Crohn’s disease approximately 15 years ago. He is currently taking prednisone and sulfasalazine, and reports feeling well. He says he still occasionally has watery diarrhea, but denies fever, abdominal pain, or weight loss. He had a colonoscopy 1 year ago which demonstrated a few transmural inflammatory lesions in his descending colon. The most important management of this patient is

  A. increase prednisone dose
  B. increase sulfasalazine dose
  C. prophylactic colectomy
  D. surveillance barium enema every year
  E. surveillance colonoscopy every year
Explanation:

The correct answer is E. The most important recommendation for this patient is a surveillance colonoscopy every year in an effort to detect colon cancer early. Although the risk of colon cancer in Crohn’s disease is much less than in ulcerative colitis, the risk increases significantly with involvement of the colon, and if the disease has been present for more than 10 years. This patient is at increased risk for colon cancer because he has had Crohn’s disease for at least 15 years and has evidence of colon involvement. A colonoscopy is recommended because it is superior at detecting small lesions and biopsy of suspicious lesions can also be performed simultaneously.

An increase in prednisone dosage (choice A) is not indicated in this patient at this time. He reports feeling well and has only rare episodes of diarrhea. Medication adjustments should be made based on the patient’s symptoms.

An increase in sulfasalazine dosage (choice B) is not indicated in this patient at this time. He reports feeling well and has only rare episodes of diarrhea. Medication adjustments should be made based on the patient’s symptoms.

Prophylactic colectomy (choice C) is not indicated or recommended for this patient. Many patients with Crohn’s disease who have extensive colitis undergo colectomy early in the course of disease to relieve persistent symptoms. This patient does not have severe symptoms nor does he have prior colonoscopy findings of dysplasia to warrant a colectomy. Prophylactic colectomy is often recommended for patients with ulcerative colitis with long standing colitis due to the increased risk of colon cancer.

Surveillance barium enema (choice D) is not the best recommendation for this patient. He has had Crohn’s disease for over 10 years and is at increased risk for developing colon cancer. A barium enema is not as sensitive or specific for the detection of early colon cancer. Colonoscopy is recommended because it is superior at detecting small lesions and biopsy of suspicious lesions can also be performed simultaneously.

 

A 37-year-old woman comes to the office because of a “burning sensation” in the chest for the past 3 months. The “burning” typically begins in the “upper stomach and travels up to the neck.” The symptoms worsen when she lies down to go to sleep. She is a chef at a local American restaurant, has 3 children, and has been married for 12 years. She “tries” to eat a healthy diet, but it is difficult because she is around food all day and night. She has no chronic medical conditions, takes no medications, and does not drink alcohol or caffeine-containing beverages. She recently quit smoking. Her temperature is 37.0 C (98.6 F), blood pressure is 120/80 mm Hg, pulse is 65/min, and respirations are 14/min. Physical examination is unremarkable. An electrocardiogram is unremarkable. A complete blood count and metabolic profile are normal. Serologic testing for H. pylori is negative. The most appropriate next step is to

  A. order ambulatory esophageal pH testing
  B. order an upper gastrointestinal barium radiograph
  C. recommend elevation of the head of bed and avoidance of food before bedtime
  D. schedule an upper endoscopy
  E. schedule esophageal manometry
Explanation:

The correct answer is C. This patient complains of the classic symptoms of gastroesophageal reflux disease (GERD). Reflux disease is usually worse at night because the recumbent position allows gastric acid contents to go up into the esophagus. Since all of the tests ordered in the case were normal, you should first recommend non-pharmacologic therapy before continuing with further diagnostic studies. Elevation of the head of bed, avoiding eating before bed, and avoiding alcohol, tobacco, chocolate, and caffeine should all be recommended. Alcohol, tobacco, chocolate, and caffeine all lower the lower esophageal sphincter pressure leading to gastric reflux. If these measures are ineffective, pharmacologic therapy with a H2 blocker such as cimetidine, famotidine, or ranitidine is indicated. For more severe symptoms, a proton pump inhibitor, such as omeprazole or lansoprazole, is indicated.

Ambulatory esophageal pH testing (choice A) is usually reserved for patients who fail nonpharmacologic and pharmacologic management.

An upper gastrointestinal barium radiograph (choice B) is useful in detecting esophageal rings or strictures, which typically present with dysphagia. This patient complains of heartburn, not dysphagia.

An upper endoscopy (choice D) is usually indicated only after the failure of nonpharmacologic and pharmacologic management for GERD and when a patient has GERD for >5 years, and upper endocsopy is recommended to screen for Barrett’s metaplasia. However, it is not indicated at this time.

Esophageal manometry (choice E) is typically reserved for cases of GERD when surgical therapy is being considered.

 

  A 51-year-old woman with end-stage liver disease due to cryptogenic cirrhosis is being cared for by your medical team. She was recently admitted for increasing abdominal girth and confusion. She was diagnosed with cirrhosis and portal hypertension 3 years ago and has long-standing ascites and 2 previous admissions for hepatic encephalopathy. She has no allergies. Her current medications include oral lactulose, ofloxacin, spironolactone, and furosemide. Over the past few days, the team has been attempting to reduce her ascites by both repeated large-volume paracentesis and aggressive diuresis. The patient has been having four to five bowel movements daily while on lactulose. On reviewing the morning laboratory data, the following values are noted:

Day 1: Sodium 126 mEq/l, Potassium 3.2 mEq/l, BUN 20 mg/dl, Creatinine 1.1 mg/dl

Day 2: Sodium 129 mEq/l, Potassium 3.5 mEq/l, BUN 29 mg/dl, Creatinine 1.4 mg/dl

Day 3: Sodium 134 mEq/l, Potassium 4.2 mEq/l, BUN 33 mg/dl, Creatinine 1.7 mg/dl

Day 4: Sodium 142 mEq/l, Potassium 4.8 mEq/l, BUN 41 mg/dl, Creatinine 2.1 mg/dl

Day 5: Sodium 148 mEq/l, Potassium 5.2 mEq/l, BUN 55 mg/dl, Creatinine 2.9 mg/dl

The most important management is to

  A. administer sodium polystyrene sulfonate, orally
  B. bolus 500cc of normal saline
  C. discontinue the furosemide
  D. discontinue the spironolactone
  E. obtain a 12 lead electrocardiogram
Explanation:

The correct answer is D. Aggressive diuresis and fluid management is the mainstay of therapy for ascites related to liver failure. In the case of this patient, the aggressive diuresis has caused progressive renal insufficiency. The potassium sparing diuretic spironolactone is only worsening the hyperkalemia and should be promptly discontinued until the patient’s renal function returns to normal.

Administering sodium polystyrene sulfonate orally (choice A) would be indicated if a 12 lead EKG suggested evidence of hyperkalemia (QRS changes, T wave elevations). Kayexalate is a potassium binding resin used to lower serum potassium.

Bolus 500cc of normal saline (choice B) may help with the worsening pre-renal failure, but will not reverse the problem in a definitive manner.

Discontinuing the furosemide (choice C) is not the primary diuretic to address, since by keeping spironolactone, the potassium elevation will likely only worsen.

Obtaining a 12 lead electrocardiogram (choice E) is not unwise but given that the potassium is not severely elevated, discontinuing the offending diuretic is the most important concept to grasp. Blindly checking an EKG for a marginally elevated potassium misses the point of a rising creatinine and potassium in the setting of aggressive diuresis.

 

A 55-year-old woman is brought to the emergency department by her daughter because of left lower quadrant abdominal pain, anorexia, fever, and chills for the past 24 hours. Her temperature is 38.7 C (101.6 F), pulse is 110/min, and respirations are 18/min. She is awake and alert, although she appears uncomfortable. Examination shows hypoactive bowel sounds and a soft abdomen with mild voluntary guarding especially in the left lower quadrant. Digital rectal examination is significant for heme-positive stool. An electrocardiogram shows a sinus tachycardia at 110 beats per minute. There are no ST segment changes when compared with old electrocardiograms. A chest x-ray shows no acute disease. Abdominal x-ray demonstrates no air under the diaphragm and no ileus. A complete blood count, biochemical profile, cardiac enzymes, amylase, and lipase are drawn, but results are not yet available. The most appropriate initial management of this patient is to

  A. admit for intravenous antibiotics, nil per os diet, and abdominal CT scan
  B. discharge to home on a clear liquid diet and PO antibiotics
  C. immediately refer to a gastroenterologist for colonoscopy
  D. prepare the patient for immediate surgical exploration
  E. send patient for urgent cardiac catheterization
Explanation:

The correct answer is A. This patient most likely has diverticulitis, an inflamed herniation of the mucosa of the colon through the muscular layer of the bowel wall. The best way to diagnose diverticulitis is with an abdominal CT scan. The appropriate management of this patient is admission to the hospital. She should be kept NPO and given IV antibiotics.

Mild cases of diverticulitis may be treated as an outpatient with a clear liquid diet and PO antibiotics (choice B). However, this patient is too sick to be discharged and ought to be treated with IV antibiotics.

A colonoscopy (choice C) is useful to diagnose diverticulosis, but would not be used during an acute attack of diverticulitis because of risk of perforation.

Indications for urgent surgical intervention include abscess formation, severe disease, or confirmed perforation. This patient does not yet show signs of needing surgical intervention (choice D).

Abdominal pain is often the presenting sign of cardiac ischemia. However, this patient has few risk factors, no EKG changes, and does have abdominal exam findings. Therefore, cardiac catheterization (choice E) would not be indicated. You should, however, still continue to monitor her cardiac function and obtain cardiac enzymes.

 

A 37-year-old man comes to the emergency department because of the abrupt onset of crampy abdominal pain and “bright red blood oozing” from his mouth. There were no episodes of emesis preceding the hematemesis. The patient has a past medial history significant for alcoholic cirrhosis documented by liver biopsy 3 years ago. He has been poorly compliant with medications and has not been seen by a physician for over 2 years. He continues to drink 6-12 beers per day. His blood pressure is 90/40 mm Hg and pulse is 90/min. Physical examination shows scleral icterus, clear lung fields, a distended and tense abdomen with a fluid wave, and diffuse spider angiomata on his chest and abdomen. There is no asterixis. You send him for upper endoscopy, which reveals grade three esophageal varices with no active bleeding. These varices are sclerosed. He is admitted to the hospital. The most appropriate next step in management to prevent morbidity is to

  A. admit the patient to the ICU for a re-endoscopy in 48 hours
  B. begin intravenous octreotide therapy
  C. observe the patient for 48 hours and then discharge to home
  D. perform an immediate portal-systemic shunt operation
  E. transfuse the patient to a hematocrit greater that 30%
Explanation:

The correct answer is B. About 30-60% of variceal bleeding episodes stop spontaneously. Therefore, in the setting of a presumed upper gastrointestinal (UGI) bleed when an endoscopy shows varices but no active lesions, banding and intravenous somatostatin or its analogue, octreotide, are indicated. These agents are vasodilators that cause a reflex increase in splanchnic vessel tone and thus serve to decrease bleeding. This is the standard of care for GI bleeds from varices.

Admitting the patient to the ICU for a re-endoscopy in 48 hours (choice A) is not indicated as the patient appears hemodynamically stable and does not require ICU monitoring.

Observe the patient for 48 hours and then discharge to home (choice C) is not acceptable since most centers choose to re-endoscope patients prior to discharge.

Performing an elective portal-systemic shunt operation (choice D) is certainly an option in the secondary prevention of UGI bleeds due to varices. It is a highly effective option. The drawback is that the procedures are associated with a significant incidence of hepatic encephalopathy. As such, there is no indication to refer all patients with variceal bleeds for elective shunt therapy, but the option should be offered to them.

Transfuse the patient to a hematocrit greater that 30% (choice E) is a “trigger” often taught but the decision to transfuse a patient should be based upon the patient’s clinical condition and not a number. If the patient is stable and the bleeding has been controlled, there is no absolute reason why a hematocrit greater than 30 must be attained. There is in fact recent literature that suggests some critically ill patients do worse with a more aggressive transfusion strategy (Hct >30) compared to a more permissive goal (Hct >24).

 

  A 73-year-old man with emphysema comes to the clinic with complaints of food getting stuck when he swallows, which has been getting progressively worse over the last 8 months. He denies problems swallowing liquids and thinks he has lost about 5 pounds. He used alcohol heavily for many years but quit drinking 10 years ago. He still smokes 1 pack of cigarettes per day and has done so since age 20. He uses albuterol, steroid inhalers and theophylline. His blood pressure is 123/73 mm Hg, pulse is 87/min, and respirations are 20/min. Physical examination reveals bilateral scattered wheezes in the lungs. A chest x-ray shows hyperexpansion and no nodules. The most appropriate next step in management is to

  A. order a barium esophagram
  B. order an esophageal manometry
  C. order an esophageal pH probe
  D. treat with omeprazole and follow up in 3 months
  E. treat with ranitidine and follow up in 3 months
Explanation:

The correct answer is A. This patient most likely has an esophageal squamous cell carcinoma (the most common type of esophageal malignancy). In any patient with dysphagia that is progressive for only solids, it suggests a growing and obstructive lesion. The history of tobacco and alcohol use, puts this person at a much higher risk of carcinoma. The two ways to diagnose this are a barium swallow study, which will show the mucosal mass, or an upper endoscopy study to directly visualize and biopsy the lesion.

Esophageal manometry (choice B) is used to evaluate dysphagia caused by motility disorders. These typically present with dysphagia for solids and liquids and may or may not be progressive.

A pH probe (choice C) is used to evaluate esophageal reflux disease, which does not in itself typically cause dysphagia, but over long periods of time will increase the risk of esophageal adenocarcinoma.

Both omeprazole (choice D) and ranitidine (choice E) are used to treat symptoms of gastroesophageal reflux disease and would not address his dysphagia. Furthermore, waiting 3 months to see the patient again would be inappropriate.

 

A 36-year-old woman comes to the office because of a 3-day history of “yellow skin,” fever, and abdominal pain. The pain is mostly present in the right upper quadrant. However she sometimes feels it in her right shoulder. She has had several similar episodes in the past, but they were not accompanied by fever, and skin discoloration. She is married and has 3 children, none of whom are sick. Her temperature is 39.3 C (102.7 F), blood pressure is 110/70 mm Hg, pulse is 70/min, and respirations are 20/min. Physical examination shows right upper quadrant tenderness. She has the “chills”, but she continues to breathe normally during right upper quadrant palpation. Laboratory studies show

The most likely diagnosis is

  A. acute cholangitis
  B. acute cholecystitis
  C. acute hepatitis
  D. acute pancreatitis
  E. biliary colic
Explanation:

The correct answer is A. This patient has fever, jaundice, and right upper quadrant abdominal pain, Charcot’s triad, which is usually diagnostic of acute cholangitis. This typically occurs because of a stone impacted within the common bile duct. Blockage of this duct results in cholestatic jaundice, hence the elevated bilirubin, and high alkaline phosphatase. Gram-negative bacteria penetrate into the biliary ducts, and cause inflammation with leukocytosis and fever. The pain is due to gallbladder distention. The treatment involves antibiotics and surgery.

Acute cholecystitis (choice B), is inflammation of the gallbladder and the obstruction of the cystic duct by a gallstone. Symptoms include right upper quadrant pain, a mild fever, and possibly nausea and vomiting. Physical examination shows right upper quadrant pain with inspiratory arrest during palpation, (Murphy’s sign). Mild jaundice may occur. The leukocyte count is elevated. Treatment includes antibiotics and surgery.

Acute hepatitis (choice C), manifests with mild right upper quadrant abdominal pain, nausea, anorexia, and a low-grade fever. Serum alanine aminotransferase and aspartate aminotransferase are markedly elevated, helping to distinguish this from cholangitis.

Acute pancreatitis (choice D), typically presents with intense midepigastric pain, usually radiating to the back, fever, nausea, and vomiting. Very high levels of amylase and lipase will support the diagnosis. The treatment usually involves insertion of a nasogastric tube, intravenous fluids, and electrolyte replacement.

Biliary colic (choice E), is episodic right upper quadrant abdominal pain that radiates to the back, and may be associated with nausea and vomiting. It often occurs after a meal. It is caused by a transient blockage of the cystic duct with a gallstone. It is not usually associated with fever or jaundice.

 

  You are called to see a patient with end-stage liver disease secondary to hepatitis C obtained from injection drug abuse. He reports that he has experienced increasing abdominal girth for the last 2 weeks. He also notes that his urine output has been minimal for the last 3 days, producing approximately 30 cc of urine each day. His temperature is 37 C (98.6 F), blood pressure is 95/60 mm Hg, pulse is 70/min, and respirations are 19/min. Physical examination reveals scleral icterus, huge abdominal distention with bulging flanks, and a fluid wave. His lower extremities have 2+ edema. Laboratory studies show:

Sodium 128 mEq/dL
Potassium 4.8 mEq/dL
Chloride 98 mEq/dL
Bicarbonate 21 mEq/dL
BUN 28 mg/dL
Creatinine 3.2 mg/dL

 

Urinalysis
Color Clear
Specific gravity 1.020
Osmolality 55 mOsmol/kg
Leukocyte esterase Negative
Nitrite Negative
Protein Negative
Blood Negative
Microscopic Few hyaline casts
Urine Sodium 4 mEq/L

In an effort to increase urine output, you perform a therapeutic paracentesis and provide a fluid challenge with 500 ml normal saline. Urine output does not improve. He is “so sick of all of this” and wants to know what is the most effective treatment. He should be told that his condition can be most effectively managed with

  A. continued fluid resuscitation with normal saline
  B. intravenous albumin therapy
  C. liver transplantation
  D. renal dose dopamine therapy
  E. treatment with furosemide
Explanation:

The correct answer is C. This patient has hepatorenal syndrome (HRS). The treatment is a liver transplant. It is seen in end-stage liver disease in patients with chronic liver disease more often than in acute fulminant hepatic failure. It is sometimes difficult to differentiate from prerenal azotemia. Typically, the urine sodium in prerenal azotemia is less than 20 mEq/L, while the urine sodium with hepatorenal syndrome is less than 10 mEq/L. Urine output will not increase with a fluid challenge in HRS while it will in prerenal azotemia. The management of HRS should include removal of ascites, and an attempt to increase urine output and a small fluid challenge with normal saline or albumin. Furosemide can be tried but it likely will not work. The prognosis is very poor unless the patient receives a liver transplant. The kidneys of these patients are normal and will begin to work with a new liver.

Fluid resuscitation (choice A) has been tried without success already in this case. Continued aggressive hydration of this patient will likely increase his extravascular volume (e.g., ascites and lower extremity edema) without increasing his urine output.

Resuscitation with albumin (choice B) is thought to be helpful in expanding intravascular volume in cirrhotic patients since their underlying problem is low oncotic pressure in the intravascular space. Albumin might help to temporarily bring fluid into the vessels but will likely also eventually become extravascular. However, this patient is likely not prerenal, so volume expansion is unlikely to be effective.

Renal dose dopamine (choice D) is controversial. It is thought to work by dilating renal vasculature and increasing blood flow to the kidneys. At low doses, dopamine acts selectively in renal vasculature, while at higher doses it acts non-selectively like norepinephrine. Studies have not shown renal dose dopamine to be effective but it is still fairly widely used despite inconclusive evidence.

Furosemide (choice E) is part of the management of cirrhotic patients as a way to decrease ascites and decrease peripheral edema. In HRS, furosemide can be attempted but it will likely not work. The most effective management of this patient with HRS is clearly liver transplant.

 

A 64-year-old woman comes to the emergency department with a 36-hour history of diffuse abdominal pain, abdominal fullness, nausea, and vomiting. She has no appetite and is unable to eat or drink secondary to nausea and vomiting, which is bilious in color. She passed loose brown stool earlier today. She denies any bright red blood per rectum or bloody vomitus. Her past medical history is notable for endometrial cancer 4 years ago treated with surgery and radiation. The patient denies ever experiencing similar symptoms in the past. Her temperature is 37.0 C (98.6 F), blood pressure is 110/70 mm Hg, pulse is 100/min, and respirations are 16/min. She has a moderately distended abdomen with diffuse tenderness on palpation. There is no rebound tenderness or guarding. Bowel sounds are high-pitched. There is no occult blood on rectum examination. Initial laboratory studies show:

The next most appropriate step to confirm the diagnosis is to obtain

  A. a CT scan of the abdomen
  B. plain films (supine abdomen x-ray and upright chest and abdomen x-ray)
  C. a right upper quadrant ultrasound
  D. serum amylase and lipase
  E. serum lactate
Explanation:

The correct answer is B. This patient most likely has a small bowel obstruction as evidenced by prior abdominal surgery, a diffusely tender abdomen, high-pitched bowel sounds, and nausea and vomiting. Small bowel obstruction usually results from either mechanical blockage or paralytic ileus. Postoperative adhesions or an incarcerated inguinal hernia are the most common causes of a mechanical small bowel obstruction. This patient is at risk for bowel obstruction given her prior abdominal surgery and radiation, thus putting her at increased risk for adhesion and stricture formation. Patients with small bowel obstruction typically present with a distended, tender abdomen, nausea and vomiting, and high-pitched bowel sounds. The presence of peritoneal signs suggests infarcted bowel or perforation. Laboratory tests may reveal a normal or elevated white blood cell count, elevated hematocrit secondary to hemoconcentration, and low potassium and chloride secondary to vomiting. Blood urea nitrogen and creatinine may be elevated secondary to prerenal azotemia. Plain films are very helpful in diagnosing small bowel obstruction and should be part of the initial management of the patient. X-rays of the abdomen commonly reveal gas-filled, distended loops of bowel with air-fluid levels. An upright chest x-ray is needed to evaluate for possible bowel perforation which is demonstrated by the presence of free air under the diaphragm.

A CT scan of the abdomen (choice A) is rarely needed to diagnose a small bowel obstruction and can worsen a patient’s symptoms, especially in the setting of a complete small bowel obstruction. In complete obstruction, administering contrast from above will add to the degree of fluid and edema proximal to the obstructed bowel. A CT scan of the abdomen may be helpful later in a patient’s course. For example, it may be necessary to obtain a CT scan to exclude the possibility of a tumor compressing bowel that leads to obstruction. This imaging modality, however, is not part of the immediate management.

A right upper quadrant ultrasound (choice C) is used to image the liver, gallbladder, and ducts. The patient does not have any abdominal symptoms or pain localized to this region to warrant starting with this imaging modality. An uncommon but important cause of small bowel obstruction in elderly patients is gallstone ileus. In this setting, a gallstone erodes through the gallbladder wall and into the small bowel, causing intraluminal obstruction. Typically, patients will give a history of right-sided upper abdominal pain at the onset of their symptoms and may demonstrate air in the biliary tree on ultrasound. This patient, however, does not have any symptoms to suggest the presence of gallstones.

An elevated serum amylase and lipase (choice D) is useful in diagnosing pancreatitis. Pancreatitis usually presents with epigastric pain radiating to the back, as well as nausea and vomiting. The two leading causes of pancreatitis are alcohol abuse and gallstones. This patient’s presentation and exam are not classic pancreatitis as described above.

An elevated lactate (choice E) is frequently elevated in the setting of hemorrhage, shock, sepsis, and hypoxia. This patient has an elevated anion gap of which suggests the presence of a lactic acidosis. An elevated lactate, however, may suggest many processes as outlined above and thus would not be helpful in confirming the diagnosis of small bowel obstruction.

 

A 53-year-old man is admitted to the hospital from the emergency department because of worsening confusion. He is brought in by a friend who reports that the patient has “liver disease”, has been drinking lately, and has not been taking his medications. The friend tells you that he has gotten progressively more confused over the past few days. She only knows a vague history but thinks the patient has “cirrhosis”. She does not think the patient has had a recent fall, even though he has not been without alcohol for any appreciable length of time. His temperature is 37.0 C (98.6 F), blood pressure is 120/70 mm Hg, and pulse is 100/min. He has deep scleral icterus and his skin is jaundiced. His lungs are clear, cardiac exam is normal, and he has a distended abdomen with shifting dullness. He is alert to person only and his neurological exam is normal with the exception of the inability to perform finger to nose touching and heel to shin maneuvers. He has asterixis. Laboratory studies show:

The most likely cause of his confusion is

  A. acute hyponatremia
  B. ascending cholangitis
  C. hepatic encephalopathy
  D. metabolic acidosis
  E. subdural hematoma
Explanation:

The correct answer is C. The most important concept to understand from this question is what complications affect cirrhotics. The evaluation of altered mental status in the emergency department is a complex topic, but one of the most useful and essential components of this evaluation is the history. When the history is given of a cirrhotic “not taking his medications,” an understanding that encephalopathy is one of the most likely diagnosis should come immediately to your mind.

Acute hyponatremia (choice A) is not equivalent to the serum Na of 129mEq/L. Acute signs of hyponatremia are seen when the serum sodium falls more than 12mEq/L in less than 24 hours. The signs of such an illness involve nausea, vomiting, confusion, and focal neurological findings related to brain edema.

Ascending cholangitis (choice B) is not supported by your physical exam or by physical findings such as the classic Charcot triad (fever, right upper quadrant pain, and jaundice).

Metabolic acidosis (choice D) is not supported by the data. The patient does have a low bicarbonate level, but his pH is not known. It is likely acidemic but this is not equivalent to inferring that the patient is suffering confusion as a result of the acidemia.

Subdural hematoma (choice E) is not the most likely diagnosis in this noncompliant, cirrhotic patient. The most common cause of SH is tearing of the bridging veins suffered during a trauma. It is certainly possible, however, since hepatic encephalopathy occurs in more than half of all cirrhotics with severe impairment of liver function, and since the friend tells you that he has not been taking his medications, this patient’s confusion is most likely due to hepatic encephalopathy.

 

  A 60-year-old man with diabetes and hypertension comes to the clinic because his wife is worried that his skin is turning yellow. The patient’s wife reports that she first noticed the skin changes about 1 month ago and now she says “even his eyes look bright yellow!” He drinks a case of beer a week and smokes 2-3 packs of cigarettes a week. He says he has been feeling well and denies abdominal pain, nausea, or vomiting. Vital signs are normal. He is a thin male and the abdominal examination is normal. Laboratory studies show:

The most appropriate test at this time is

  A. CT of abdomen and pelvis
  B. endoscopic retrograde cholangiopancreatography (ERCP)
  C. mesenteric angiography
  D. serum CA19-9
  E. upper gastrointestinal barium study
Explanation:

The correct answer is A. The presentation of painless jaundice is highly suspicious for a pancreatic head mass and in particular adenocarcinoma of the pancreas. Adenocarcinoma of the pancreas accounts for more than 90% of pancreatic malignancies and jaundice is present in about 65% of patients. Risk factors for pancreatic adenocarcinoma include smoking and diabetes. The best initial evaluation for pancreatic masses is by CT of the abdomen and pelvis.

Endoscopic retrograde cholangiopancreatography (ERCP) (choice B) is not the best initial test to evaluate for pancreatic masses because it primarily evaluates the biliary duct system in the liver and pancreas. It will not define a mass that does not involve the biliary ducts. An ERCP may show a discrete stricture in the main pancreatic duct with proximal dilatation.

A mesenteric angiography (choice C) is not the best initial test to evaluate for pancreatic masses because it primarily evaluates the vascular structures that supply and drain the pancreas and abdominal organs. Angiography can be useful preoperatively because it may show displacement or encasement of the pancreatic or duodenal arteries by a mass. The venous phase is also useful if the superior mesenteric vein or splenic vein is occluded due to tumor extension.

Serum CA19-9 (choice D) is not the best initial test to evaluate for pancreatic adenocarcinoma. It is a tumor marker that has been associated with adenocarcinoma of the pancreas. It is not useful as a screening test, but it has a high sensitivity and specificity as a marker for recurrent disease or metastases after the primary pancreatic tumor is resected.

An upper gastrointestinal barium study (choice E) is not the best initial test to evaluate for pancreatic masses. It is a useful study to evaluate for mucosal lesions within the esophagus, stomach, and small bowel. In the setting of pancreatic malignancy or other masses, the upper gastrointestinal barium study may show a widened loop or an “inverted 3 sign” caused by abnormal indentation of the pancreas along the medial aspect of the duodenum.

 

 

An asymptomatic previously healthy 60-year-old man comes to the office because he is found to have a liver mass. Recently, he had epigastric and right upper quadrant pain, which was investigated by means of ultrasound. Sonography demonstrated a lesion in the right lobe of the liver, but no gallstones or evidence of cholecystitis. Further investigations by means of endoscopy revealed gastritis from Helicobacter pylori, for which he was treated. He is concerned about this liver mass and hence, comes to the office. The liver mass is described as an 8-cm solitary lesion within the right lobe of the liver. No enterohepatic biliary ductal dilatation was noticed. A CT scan of the abdomen performed with contrast demonstrated a progressive peripheral to central prominent enhancement and a central hypodense region. An MRI shows a dense T2 weighted image. The most appropriate next step in the management of this patient’s liver lesion is

  A. celiac arteriography
  B. observation
  C. percutaneous needle biopsy
  D. radiation therapy
  E. resection
Explanation:

The correct answer is B. The characteristic lesion described in this patient fits in well with a diagnosis of a cavernous hemangioma. A cavernous hemangioma is a hamartomatous transformation blood vessel. This can be classically diagnosed with a contrast CT or a T2 weighted image on MRI. A radiolabeled red blood cell scan can diagnose a hemangioma, but is not usually necessary. Observation is typically indicated, as rupture is rare.

Celiac arteriography (choice A) would be able to diagnose a hemangioma, but is not necessary in the workup of this patient.

A percutaneous needle biopsy (choice C) is contraindicated because of the risk of bleeding.

Radiation therapy (choice D) has no established role in the treatment of a hemangioma.

Resection (choice E) is indicated for a rapidly expanding hemangioma, for a symptomatic lesion, or if the diagnosis is uncertain. Enucleation is usually carried out in these cases.

 

  A 48-year-old woman comes to the emergency department with right upper quadrant pain. Except for minor epigastric and right upper quadrant discomfort in the past few months, she reports being in good health. She never sought medical evaluation, but did take over-the-counter antacids. Now she complains of right upper quadrant pain for the past 4 hours that started abruptly during the night and woke her up from sleep. Since then, the pain has been persistent in the right upper quadrant and is progressively getting worse. Her temperature is 37.9 C (100.2 F), blood pressure is 140/80 mm Hg, and pulse is 94/min. Chest auscultation reveals slightly diminished breath sounds in the base of the right lung. Abdominal examination reveals a soft, distended abdomen with diffuse discomfort, localized to the right upper quadrant with a positive Murphy’s sign. Laboratory studies show a leukocyte count of 16,000/mm3. Her serum bilirubin is 1.4 mg/dL. The remainder of the complete blood count, metabolic panel, and liver function tests are within normal limits. A clinical diagnosis of acute cholecystitis is made and the patient is referred for sonography. Ultrasonography of right upper quadrant demonstrated no gallstones, but gallbladder wall thickening with peripheral cystic fluid. You diagnose her with acute cholecystitis and admit her to the hospital for treatment with intravenous antibiotic therapy. Three hours after admission to the hospital, you are called to the floor as she is complaining of severe abdominal pain, which got worse since admission. On examination, her vitals are unchanged, but her abdominal examination reveals voluntary guarding, right upper quadrant pain, and board-like rigidity of the abdomen. The most appropriate next step is to order

  A. an abdominal x-ray, erect and supine
  B. a CT scan of the chest
  C. a CT scan of the abdomen and pelvis
  D. a hepatobiliary nuclear scan
  E. an upper gastrointestinal endoscopy
Explanation:

The correct answer is A. This patient presented with sudden onset of right upper quadrant pain associated with nausea and vomiting. She has had right upper quadrant and epigastric pain before, associated with her food intake. These symptoms are classical for a perforated peptic ulcer. In a perforated peptic ulcer, a patient can still have right upper quadrant localized tenderness, a thickened gallbladder wall, and pericholecystic fluid from the perforated ulcer. Hence, with any abdominal pain associated with signs, abdominal x-rays both erect and supine are very essential in the initial evaluation to rule out any free air. Worsening of the symptoms, development of board like rigidity, and voluntary guarding are signs of peritonitis from a perforated peptic ulcer.

A CT scan of the chest (choice B) is not indicated in this patient, as there were no suspected pulmonary symptoms and no signs.

A CT scan of the abdomen and pelvis (choice C) might reveal free air, free leakage of the contrast from the perforated peptic ulcer, or infiltration of the mesentery and omentum in the upper quadrant, suggestive of perforated peptic ulcer. But, a simple abdominal x-ray is often essential to diagnose free air, before performing the CT scan of the abdomen and pelvis.

A hepatobiliary scan (choice D) has a high sensitivity for diagnosing acute cholecystitis, but is not useful in a perforated peptic ulcer.

An upper gastrointestinal endoscopy (choice E) is essential as a primary investigative tool in elective patients. In patients with peritonitis and a suspected perforated peptic ulcer, an upper GI endoscopy is not the first investigational choice.

 

 

  A 48 year-old man with hypertension and cirrhosis is brought to the emergency department by his wife because of hematemesis. This morning he woke up feeling nauseated and vomited “coffee ground” looking material. He then ate his breakfast and afterwards, vomited bright red blood. His medications include atenolol, ranitidine, and folate. The most appropriate next step in evaluating this patient is to

  A. assess airway and respiratory status
  B. give him an emergent blood transfusion
  C. insert a nasogastric tube
  D. perform upper endoscopy and sclerotherapy
  E. prepare him for immediate laparotomy
Explanation:

The correct answer is A. For all patients that present in an emergency situation, the same basic approach should be applied. This approach, the ABCs (airway, breathing, and circulation) are a reminder that these basics must always be assessed first; the so-called primary survey. Once this is accomplished, more detailed interventions can be taken based upon additional findings. A patient that has vomited blood must have their airway and breathing assessed to ensure that there is no compromise that is life-threatening.

Emergent blood transfusion (choice B) may be appropriate after laboratory data are obtained that show a low hematocrit or if there are clinical signs suggestive of profound hypovolemia or shock.

Insertion of a nasogastric tube (choice C) may be appropriate, and is in fact likely to be appropriate, after the initial primary survey is completed. This allows for decompression of any remaining blood in the stomach and serves to evacuate any ongoing losses to prevent additional episodes of emesis.

An upper endoscopy and sclerotherapy (choice D) will likely be the treatment option of choice once the patient is stable and fully evaluated. Most common causes of upper GI bleeding are amenable to diagnosis and treatment by upper endoscopy.

Immediate laparotomy (choice E) may be indicated if the source of the bleeding can only be corrected surgically. These procedures are however NOT diagnostic.

 

 

A 58-year-old homeless man is brought to the emergency department with severe hematemesis. He has a history significant for severe alcohol abuse and significant esophageal varices with bleeding in the past. You notice in his old chart that it was recommended that he take a multivitamin, folate, and thiamine. His blood pressure is 100/50 mmHg, pulse is 105/min, and respiratory rate is 26/min. Physical examination shows coarse breath sounds and a protuberant abdomen. Nasogastric lavage yields fresh blood. Given that you strongly suspect another variceal bleed, the most appropriate next step in the management of this patient is

  A. administration of amiodarone, intravenously
  B. administration of nadolol, orally
  C. administration of nitroglycerin, intravenously
  D. administration of octreotide, intravenously
  E. administration of phenylephrine, intravenously
Explanation:

The correct answer is D. Octreotide, a synthetic somatostatin analog, is used in the acute management of variceal bleeding. It decreases variceal bleeding by decreasing splanchnic blood flow and decompressing the portal system.

Amiodarone (choice A), an anti-dysrhythmic agent, has no role in the management of a variceal hemorrhage.

Oral nadolol (choice B) has been shown to be of significant benefit in the primary and secondary prevention of variceal bleeding, but plays no role in its acute management. A beta antagonist should also be used in the setting of acute hemorrhage with extreme caution.

Intravenous nitroglycerin (choice C) plays no role in the acute management of a variceal bleed. This is despite evidence that nitrates play a role in the primary and secondary prevention of variceal bleeding in combination with beta antagonists. Vasodilators such as nitroglycerin are also contraindicated with hemorrhage.

Phenylephrine (choice E), an alpha agonist vasopressor, has no role in the management of variceal hemorrhage. This patient’s hemodynamics also do not warrant vasopressor therapy.

 

A 67-year-old woman with peripheral vascular disease, bilateral leg claudication, and hypertension comes to the clinic because of nausea and severe, diffuse abdominal pain that she rates as 7/10 in intensity for the past 2 days. The pain is related to meals, particularly lunch. She has smoked a pack of cigarettes per day for the past 30 years. The patient has a temperature of 36.1 C/(97 F) with a pulse of 80/min and a blood pressure of 120/80 mm Hg. Abdominal examination demonstrates normal bowel sounds, no tenderness, and no hepatosplenomegaly. Laboratory studies reveal a leukocyte count of 4,000/mm3 and a hematocrit of 47%. You should be immediately suspicious of

  A. acute appendicitis
  B. acute cholecystitis
  C. malingering
  D. mesenteric ischemia
  E. ulcerative colitis
Explanation:

The correct answer is D. Mesenteric ischemia , although uncommon, must remain on the differential diagnosis of abdominal pain. The hallmark of mesenteric ischemia is pain out of proportion to physical exam findings. Mesenteric ischemia is especially likely in a patient with known vascular disease and a history of cigarette smoking. The next diagnostic step is a mesenteric angiogram. The superior mesenteric artery is the most often compromised vessel.

Acute appendicitis (choice A) may present with atypical symptoms in the elderly, but is usually present with a fever or elevated white blood cell count. Appendicitis is uncommon in the elderly.

Acute cholecystitis (choice B) should present with right upper quadrant pain and a positive Murphy’s sign.

Malingering (choice C) should be considered on the differential diagnosis for any patient complaint. It is, however, diagnoses of exclusion that must be entertained only when an extensive diagnostic work up is completed and is not suggestive of a disease process.

Ulcerative colitis (choice E) should present with diarrhea, constipation, heme positive stools, and abdominal pain.

 

A 39-year-old man comes to the office because of “gnawing” abdominal pain and diarrhea for the past 2 months. He states that the pain is worst about 3 hours after a meal and it often wakes him at night. He says, “surprisingly, the pain is relieved by food.” He takes a nonsteroidal antiinflammatory drug every couple of weeks for a headache or back ache, does not smoke cigarettes, and has a couple of glasses of wine on the weekends. He vaguely recalls that his father and brother have had similar symptoms in the past. Physical examination shows epigastric tenderness, midway between the xiphoid process and the umbilicus. There is no rebound tenderness. You prescribe amoxicillin, bismuth, and metronidazole, and tell him to return in 2 months. He returns for his follow-up appointment and says that his diarrhea is still present and that the abdominal pain has not decreased in intensity or quality. Physical examination is unchanged. Laboratory studies show:

At this time the most appropriate management is to

  A. measure serum gastrin levels
  B. measure serum secretin levels
  C. order a CT scan of the abdomen
  D. order an MRI of the abdomen
  E. order an ultrasound of the abdomen
Explanation:

The correct answer is A. This patient most likely has Zollinger-Ellison syndrome (ZES), which is typically characterized by peptic ulcers, increased gastrin secretion, and a tumor in the pancreas (gastrinoma). Many patients with ZES have multiple endocrine neoplasia I (MEN I), which is an autosomal dominant disorder consisting of tumors in the parathyroid gland, pancreas, and pituitary gland. Individuals with ZES often have multiple recurrent or treatment-resistant peptic ulcers, hypercalcemia, diarrhea, and a family history of pancreatic, parathyroid, or pituitary tumors. The first study used to evaluate an individual for ZES is serum gastrin levels. Individuals with ZES typically have markedly elevated levels of serum gastrin.

The first study used to diagnosis ZES is measuring serum gastrin levels, not serum secretin levels (choice B).

A CT scan of the abdomen (choice C) or an ultrasound (choice E) may be helpful in localizing the gastrinoma, after establishing the diagnosis with elevated serum gastrin levels.

An MRI (choice D) is useful in identifying hepatic metastases of gastrinomas. However, it is not the most appropriate first step in establishing the diagnosis of a gastrinoma.

 

A 44-year-old man comes to the emergency department complaining of severe abdominal pain and coffee ground vomitus. The patient is a busy financial executive who reports that over the past few months he has had increasing abdominal pain associated with eating. The patient reports some mild reflux of acid in between meals but has no prior episodes of emesis. This morning, on his way to the office he developed the acute onset of mid-epigastric pain associated with nausea. Ten minutes later, he vomited coffee ground-like material. An upper endoscopy is performed and the patient is found to have a large ulcer in the first portion of the duodenum. There is no visible vessel or active bleeding seen. He is admitted to the hospital. You go to examine him and he is awake and alert and wants to know about his disease. At this time the most correct statement about this patient’s condition is:

  A. The lesion is benign and is not the cause of the bleed
  B. The lesion is premalignant and likely caused the bleeding
  C. The lesion must be treated or the bleeding will likely recur
  D. The lesion must be treated but the bleeding will still recur
  E. There is no effective treatment for this lesion
Explanation:

The correct answer is C. The cause of more than 90% of duodenal ulcers is infection with Helicobacter pylori. The treatment for these ulcers, when non-bleeding, is antibiotic therapy. When they have bled, in addition to all of the appropriate therapy for a bleeding patient, they still need to be tested and treated for infection. Without the proper treatment for the infection, their ulcers will worsen or they will develop new ones and they will be at continued risk for bleeding.

The lesion is benign and is not the cause of your bleed (choice A) is incorrect. The lesion identified in the vignette is the classical lesion for such episodes of bleeding.

The lesion is premalignant and likely caused your bleeding (choice B) is incorrect. Premalignant lesions that are associated with upper GI bleeding are almost solely confined to the stomach.

The lesion must be treated but bleeding will still recur (choice D) is incorrect because therapy for duodenal ulcers is curative and therefore will attenuate, if not abrogate any future risk of rebleeding.

There is no effective treatment for this lesion (choice E) is incorrect. Very effective treatment for this infection exists and it includes metronidazole, tetracycline, and bismuth orally for two weeks. In many centers, an oral macrolide antibiotic such as clarithromycin and a proton pump inhibitor such as omeprazole are becoming standard therapy.

 

  A 47- year-old man comes to the emergency department because he is “not feeling well and his abdomen is bloated and painful.” He denies any previous medical history. He reports that he has had similar episodes in the past, which resolved spontaneously. This episode started 12 hours ago, when he started feeling discomfort and pain in the abdomen. He has not passed flatus since then. He is feeling nauseous. His temperature is 38.1 C (100.6 F), blood pressure is 146/80 mm Hg, pulse is 94/min, respirations are 16/min, and oxygen saturation is 98% on room air. His abdomen is distended with fullness in the right upper quadrant and empty in the left lower quadrant. He has marked tenderness in the left lower quadrant. Rectal examination is positive for occult blood. His leukocyte count is 16,000/mm3. A chest x-ray is unremarkable. An abdominal x-ray shows a distended colonic loop pointing towards the left lower quadrant. The most appropriate next step in management is to

  A. admit the patient for a colonoscopy on the following day
  B. obtain an emergency surgical consult and prepare the patient for the operating room
  C. order a barium enema
  D. order a CT scan of the abdomen
  E. order tap water enemas
Explanation:

The correct answer is B. This patient presents with the classic symptoms and signs of sigmoid volvulus. An abdominal x-ray is usually diagnostic. The next step in the management depends upon the clinical examination. In the absence of peritoneal signs, flexible sigmoidoscopy helps in the detorsion of the colon. When the volvulus is not treated for a long time, part of the sigmoid colon can become ischemic. In the presence of a tender abdomen associated with tachycardia, fever, and a raised total leukocyte count, like in this patient, ischemia should be suspected. This patient should be taken to the operating room for immediate detorsion of the volvulus and to resect the ischemic bowel.

Patients with partial intestinal obstruction should be admitted for non-operative management to decompress the bowel and a colonoscopic examination to rule out an obstructing lesion, only in the absence of peritoneal signs. Nonoperative management (choice A) is contraindicated if the patient develops peritoneal signs or evidence of complete bowel obstruction either clinically or radiologically.

A barium enema (choice C) may be diagnostic in the management of colonic obstruction as an elective investigation. In children with intussusception, it is both diagnostic and therapeutic. In a patient with peritoneal signs, either an adult or a child, a barium enema is harmful.

A CT scan of the abdomen (choice D) is helpful to rule out diverticulitis in a patient with a similar history and physical exam, but without evidence of volvulus on the abdominal x-ray. In patients with diverticulitis, the abdomen is usually uniformly and mildly distended. In the presence of classic signs of volvulus, with a distended colonic loop in the right upper quadrant and tip pointing towards left lower quadrant, a CT scan of the abdomen is not indicated.

Tap water enemas are harmful in an acute abdomen without a definitive diagnosis (choice E).

 

  A 62-year-old man comes to the office for a periodic physical examination. He has no complaints. His past medical history is significant for mild systolic hypertension, non-insulin dependent diabetes mellitus, and atrial fibrillation. He is taking enteric-coated aspirin 81 mg daily and warfarin for his atrial fibrillation. He also reports that he is taking an herbal medicine for “strength and vitality”. Review of the herbal medicine package reveals that the medicine contains iron and vitamins. Physical examination is unremarkable. Abdominal examination is benign. Rectal examination reveals guaiac-positive stool. Rectal examination and a repeat guaiac test in the subsequent 2 days reveals guaiac-positive stools. He denies any recent alteration of bowel habits or recent loss of weight. The most appropriate next step in the management of this patient is

  A. colonoscopy
  B. observation
  C. repeat guaiac test after stopping aspirin for 1 month
  D. repeat guaiac test after stopping the iron-containing herbal medicine for 1 month
  E. repeat guaiac test after stopping warfarin for 1 month
Explanation:

The correct answer is A. The likelihood that a guaiac-based test will be positive is directly proportional to the quantity of fecal heme, which in turn is related to the size and location of the bleeding lesion. Guaiac-based tests are generally best at detecting large, more distal neoplasms. The guaiac test can be affected by dietary factors such as meat and vegetables that contain peroxidase. Iron by itself does not cause erroneous guaiac-positive stool. Iron in large quantities can yield black colored stools, which might be difficult to differentiate from positive guaiac with bluish coloration. Aspirin or warfarin slightly increases the blood loss in the stool, but by themselves do not cause guaiac-positive stools. Hence, guaiac-positive stools on 3 different occasions in a 62-year-old patient, requires colonoscopy to rule out any colonic malignancy.

Observation (choice B) is not an option when guaiac-positive stools are discovered on three different occasions.

A repeat guaiac test after stopping aspirin (choice C) is not essential; the guaiac positive stools are not caused by aspirin.

A repeat guaiac test after stopping iron containing herbal medicines (choice D) is not required, as iron by itself does not cause guaiac-positive stools.

A repeat guaiac test after stopping warfarin (choice E) is not essential, as warfarin by itself, does not cause guaiac-positive stools.

  A 47-year-old man comes to the clinic for follow-up care of his ascites and cirrhosis. He was diagnosed with cirrhosis due to hepatitis C 4 years ago that he believes that he contracted from a blood transfusion. He is anxiously awaiting liver transplantation. His only other medical history is that he has diabetes mellitus controlled with insulin. He reports to you that he avoids all alcohol consumption and takes his medications, which include spironolactone, furosemide, multi-vitamins, nadolol, and insulin. He complains, however, that his abdomen continues to “get bigger” despite the fact he limits his water intake to less that 1 liter per day. In the office his blood sugar is 198mg/dL. His physical examination is unchanged from previous visits except for more abdominal distention and 2+ lower extremity edema. In discussing his increasing ascites, he should be advised that

  A. this is because his blood sugar is poorly controlled
  B. this is due to his poor compliance with medications
  C. this is to be expected since his disease is progressive
  D. this is to be expected since he is drinking too much water
  E. this is to be expected since he is not limiting his salt intake
Explanation:

The correct answer is E. The important concept to understand from this question is that, although diuretics are effective in controlling ascites, dietary sodium consumption negates all such benefit. Any patient who you believe to be compliant with their medications that has a continued increase in their ascites, is almost certainly having too much dietary sodium (>2 gm).

This is to be expected because your blood sugar is so poorly controlled (choice A) is not relevant to the care of this patient’s ascites although it is relevant from a volume standpoint since if he is dehydrated from glucosuria, he will consume more fluids, and likely salt.

This is to be expected since you have very poor compliance (choice B) is not an appropriate comment to make to a patient and furthermore is not supported in your interview with the patient.

This is to be expected since your disease is progressive (choice C) is not an acceptable answer since the progressive nature of disease effects a patient’s mental status, bleeding risk, and infection risk, not necessarily the refractoriness of their ascites to diuresis.

This is to be expected since you are drinking too much water (choice D) is propagated throughout medicine as a cause for both edema due to CHF as well as ascites. In fact, water consumption, unless truly excessive, is irrelevant to total body fluid balance. The key determinant of sodium excretion and thus fluid excretion by the kidney is sodium intake.

 

A 48-year-old investment banker comes to the office because of a 4-month history of achy abdominal pain. He says that the pain is exacerbated by meals and he often feels very nauseous. He is generally very healthy except for some mild lower back pain for which he takes ibuprofen. He estimates that he has taken 2 over-the counter ibuprofen pills every 3 days for the past few months. He smokes a half pack of cigarettes a day and drinks a glass of wine with dinner every night. He works until 10 p.m. on weekdays and both days of the weekends. He has to take care of his children in his spare time and says that he is very “stressed out.” Physical examination shows mild epigastric tenderness. A urea breath test is positive and a barium study shows a 1.5 cm discrete crater in the antrum of the stomach with radiating mucosal folds originating from the ulcer margin. The most likely cause of this patient’s condition is

  A. alcohol consumption
  B. a bacterial infection
  C. chronic use of ibuprofen
  D. cigarette smoking
  E. psychological stress
Explanation:

The correct answer is B. This patient has a gastric ulcer that is most likely due to an infection with H. pylori. H. pylori is a Gram-negative rod that can be detected by the urea breath test, serology, a rapid urease test, or histological evaluation of a biopsy specimen. The urea breath test has a greater than 90% sensitivity and specificity for H. pylori, so a bacterial infection is the most likely cause of his condition.

Alcohol consumption (choice A) has been thought to play a role in the pathogenesis of peptic ulcer disease (gastric and duodenal ulcers), but its exact role is not proven. Since he had a positive urea breath test, H. pylori is the most likely cause of his condition, not alcohol.

Chronic use of ibuprofen (choice C) is associated with peptic ulcer disease. However, this patient does not take large amounts of ibuprofen. Also, because he had a positive urea breath test, he has an H. pylori infection. While it is possible to have an NSAID-induced gastrointestinal ulceration and a concomitant infection with H. pylori, in this case his condition is most likely caused by the bacteria.

Cigarette smoking (choice D) has been associated with peptic ulcer disease, but the exact mechanism is unknown. This patient’s positive urea breath test is consistent with an H. pylori infection. The cigarette smoking may exacerbate his condition, but it is unlikely to be the main cause.

Psychological stress (choice E) has long been associated with peptic ulcer disease, but studies on the subject have produced conflicting results. It is possible that this patient’s stress may exacerbate his symptoms but it is very unlikely that it is the cause of his condition.

 

  A 10-year-old boy is admitted to the pediatrics unit with rectal bleeding and right lower quadrant abdominal pain. He has no significant past medical history. Vital signs are: temperature 37.2 C (99 F), blood pressure 90/40 mm Hg, pulse 80/min, and respirations 11/min. The physical examination is normal. Rectal examination reveals bright red blood, but no other abnormalities. A colonoscopy extending to the ileocecal valve is normal except for a moderate amount of fresh blood. The next step in managing this patient is to order a(n)

  A. abdominal angiography
  B. nuclear medicine technetium scan
  C. sigmoidoscopy
  D. small bowel follow through
  E. upper gastrointestinal endoscopy
Explanation:

The correct answer is B. Lower gastrointestinal bleeding in a child with a negative endoscopy is suspicious for a Meckel’s diverticulum (MD). A MD occurs in 2% of the population and 2% are symptomatic. They occur 2 feet from the ileocecal valve, are usually 2 inches in length, and contain 2 types of mucosa (gastric and pancreatic). A MD is a true diverticulum from the antimesenteric border of the small bowel, and is the most common congenital abnormality of the gastrointestinal tract. It is usually asymptomatic, but may develop symptoms, usually before the age of 12. Bleeding may either be pronounced, as in this case, or present as a subtle anemia. A technetium-99m pertechnetate scan is about 90% accurate in its diagnosis. Presentation may be bleeding (50%) or obstruction (25%).

Abdominal angiography (choice A) is only useful in the unstable patient with a rising pulse and potentially, falling blood pressure. In this case, the patient is hemodynamically stable, and a nuclear medicine technetium scan is appropriate.

Sigmoidoscopy (choice C) is not necessary, as it will offer no more information than the previously performed colonoscopy.

A small bowel follow through (choice D) sometimes reveals a Meckel’s diverticulum in asymptomatic patients. It is not the study of choice in this symptomatic patient.

Upper gastrointestinal endoscopy (choice E) would not be appropriate as this patient is having lower gastrointestinal bleeding.

 

A 37-year-old woman comes to the emergency department because of a 30-minute history of vomiting reddish-brown material. She informs you that she suffers from fibromyalgia syndrome and uses a number of “pain killers” to control her pain. Her blood pressure is 120/70 mm Hg and pulse is 110/min, no orthostasis. Physical examination is unremarkable. Her extremities are cool and her capillary refill is less than 2 seconds. A nasogastric tube is passed and returns 200 cc of coffee ground material that eventually clears with normal saline lavage. The patient is then sent for endoscopy. The most likely cause of this patient’s gastrointestinal bleeding is

  A. esophageal varices
  B. esophagitis
  C. gastric neoplasm
  D. gastric ulcers
  E. Mallory Weiss tears
Explanation:

The correct answer is D. The most common causes of upper gastrointestinal (UGI) bleeds are peptic ulcer disease (PUD) (45-50%), gastritis (30%), varices (10%), and then the remainder of causes are due to other disorders such as Mallory-Weiss tears, esophagitis, and neoplasms. In any patient with a history of “pain killer” use, especially females with rheumatological conditions, the diagnosis of gastritis or gastric ulcers secondary to nonsteroidal anti-inflammatory drug (NSAID) use must be suspected. The most common cause of these two conditions is NSAID use.

Esophageal varices (choice A) are a very common cause of UGI bleeds in patients with cirrhosis. In the United States, the most common cause of cirrhosis is alcohol and hepatitis virus infection. Worldwide, Schistosomiasis is the most common cause. Since this patient has none of the above diseases, the likelihood of her having varices is almost zero.

Esophagitis (choice B) is usually due to acid reflux diseases and is not a significant cause of GI bleeding. It may lead to a premalignant condition (Barrett esophagus).

Gastric neoplasm (choice C), although accounting for a small percentage of UGI bleed patients, requires other associated findings of cancer in order to be suspected. Gastric cancer in particular is associated with early satiety, epigastric pain, palpable abdominal mass, and certain nitrate containing foods.

Mallory-Weiss tears (choice E) are small esophageal tears induced by vomiting. It should be suspected in patients who have the triad of hematemesis, alcohol abuse, and vomiting. It is not a cause of severe or prolonged or recurrent UGI bleeding.

 

  A 67-year-old man comes to the office for a follow-up visit to review the findings from a colonoscopy that was performed 2 weeks earlier. A 0.9 cm tubular adenoma was removed from his sigmoid colon. No other lesions were visualized in the colon. He has no family history of colon cancer and is very concerned when you tell him that the polyp was adenomatous. All previous colonoscopies were normal. In explaining the findings to him, you should tell him that:

  A. A chest x-ray should be performed to ensure that there are no abnormalities associated with the adenoma
  B. A colectomy should be performed to avoid the risk of developing colon cancer
  C. Colonoscopy will be required every 6 months to determine if any new polyps have formed
  D. His children should have screening colonoscopies beginning at age 25
  E. Tubular adenomas such as his have a low risk of malignant potential
Explanation:

The correct answer is E. Colonic polyps are very common in older patients, with approximately 40% of all patients at age 60 having at least 1 adenomatous polyp, (and 50% at age 70). Since there is a link between polyps and the development of malignancy, it is recommended that polyps be removed and evaluated. There are 3 types of adenomas—tubular, tubulovillous, and villous. Tubular adenomas have a low risk for malignant foci (approximately 5% risk), tubulovillous have an intermediate risk (approximately 20%), and villous have a high risk (approximately 40%).

A chest x-ray should be performed to ensure that there are no abnormalities associated with the adenoma (choice A) is incorrect. Since the case does not say that the tubular adenoma found has any malignant foci, there will not be any associated changes found in the lung.

A colectomy should be performed to avoid the risk of developing colon cancer (choice B) is inappropriate. This patient does not have a malignancy and therefore any additional treatment for this polyp is unnecessary. Prophylactic colectomies are typically recommended for patients with autosomal dominant polyposis syndrome because it is associated with an almost 100% risk of colon cancer by age 40.

Patients with a history of colonic polyps need to be followed closely with a colonoscopy every 1-3 years, not every 6 months (choice C).

Since about 40-50% of all patients in this age range will have at least 1 polyp, it is not that unusual that this 67-year-old patient has a polyp. His children should NOT have screening colonoscopies beginning at age 25 (choice D) because this is too young and the chance of them having polyps at that age is small. According to the United States Preventive Services Task Force, the recommended screening for colon cancer is an annual fecal occult blood test and/or a sigmoidoscopy every 3-5 years beginning at age 50.

    A 69-year-old man is brought to the clinic from his convalescent home, because of decreased mental status. He has a history of Alzheimer’s disease, depression, hypertension, coronary artery disease, and glaucoma. There is a “do not resuscitate” (DNR) order on the chart signed by the patient’s wife. His temperature is 37.0 C (98. 6 F), blood pressure is 110/70 mm Hg, and respirations are 16/min. Physical examination shows a distended abdomen without focal tenderness or peritoneal signs and hard stool in the rectal vault. The patient is alert and oriented only to person. An electrocardiogram reveals normal sinus rhythm with a few premature ventricular contractions (PVC). Laboratory studies are normal. A plain x-ray of the chest reveals multiple pulmonary nodules. A plain x-ray of the abdomen demonstrates a distended ascending and transverse colon measuring 20 cm with copious stool present. The next step in the evaluation of this patient is to

  A. consult with the family to consider comfort care only
  B. discharge him to the convalescent home with comfort care only
  C. order a barium enema after a preparatory enema
  D. order a CT scan of the thorax
  E. prepare him for a laparotomy
Explanation:

The correct answer is C. This patient has symptoms of fecal impaction which is common in the elderly and in debilitated patients. A preparatory enema to clean out impacted stool is necessary prior to a barium enema and is likely to be therapeutic in this patient. Subsequent barium enema could assess the cause of large bowel dilatation which is unusual in the setting of simple fecal impaction. In this case, colon cancer is a definite possibility.

A “do not resuscitate” order is not a mandate to forego appropriate minimally invasive diagnostic tests such as an enema. Hence, comfort care only (choice A) would not be appropriate as a diagnosis of a potentially easily treatable condition such as stool impaction.

A “do not resuscitate” order is not a mandate to forego appropriate minimally invasive diagnostic tests such as an enema. Discharge (choice B) is not yet warranted.

A CT of the thorax (choice D) is not necessary now as there are clearly multiple nodules on the chest radiograph.

A laparotomy (choice E) is not indicated at this point in the diagnostic workup. Moreover, surgery would only be performed in a patient with a do not resuscitate order after consult with the family or durable power of attorney.

 

  A 39-year-old man comes to the office complaining of a 3-day history of severe abdominal pain and cramps that are relieved with bowel movements. He also reports loose, watery stools two to five times per day. He has had similar symptoms in the past and recalls the first incident being nearly 12 years ago. He tells you that he has been told that he has irritable bowel syndrome. He states that he has never had any other “tests” and was only prescribed various medications, some of which seemed to have helped. On examination, he appears to be in mild distress. His temperature is 38.3 C (101.0 F). He has mild guarding in his left lower quadrant but no rebound tenderness. He is tender to direct palpation in his left lower and left middle quadrants. The most appropriate next step in this patient’s care is to

  A. prescribe corticosteroids and see the patient in two weeks
  B. prescribe loperamide and see the patient in two weeks
  C. order a stool Gram stain and culture
  D. schedule the patient for an immediate colonoscopy
  E. schedule an immediate CT scan of the abdomen
Explanation:

The correct answer is E. Although this patient carries the diagnosis of irritable bowel syndrome, his presentation on this occasion has some elements that are concerning. In particular his fever and impressive abdominal examination coupled with his diarrhea and pain raise a high suspicion for an acute abdomen of some variety. This process may or may not be related to any existing abdominal pathology that this patient may have. He requires imaging of his abdomen to rule out an acute abdominal process such as abscess, pancreatitis, appendicitis, or even colitis.

Prescribing corticosteroids (choice A) or loperamide (choice B) and seeing the patient in two weeks presumes that this presentation has a similar etiology to his past presentations. Again, the impressive abdominal examination and fever makes the likelihood of this being related simply to irritable bowel syndrome very unlikely.

A stool Gram stain and culture (choice C) would be useful, but not more so than abdominal imaging.

Arranging for an immediate colonoscopy (choice D) is not correct for two reasons. First, the bowel preparation required for a good study doesn’t make this test useful for acute situations. Secondly, the test limits visualization of possible etiologies to those that affect the large bowel.

 

 

A 79-year-old man is admitted to the hospital for a gangrenous right foot. He has a long history of peripheral vascular disease, hypertension, hypercholesterolemia, coronary artery disease, and has suffered 2 strokes. The patient’s daughter visited him at home today and noticed his foot was black. The patient is admitted to the hospital for a right, below-the-knee amputation. Over the next 48 hours the patient complains of increasing abdominal pain. His temperature is 39.8 C (103.6 F), blood pressure is 100/50 mm Hg, pulse is 120/min, and respirations are 22/min. Physical examination shows a diffusely tender and distended abdomen and his right foot is unchanged. Stat blood work is drawn and shows:

The diagnostic procedure most likely to establish the diagnosis is

  A. abdominal radiographs flat and upright
  B. chest films flat and upright
  C. CT scan of the abdomen
  D. exploratory laparotomy
  E. ultrasound of the abdomen
Explanation:

The correct answer is D. This patient has an ongoing abdominal catastrophe, likely an ischemic bowel or bowel perforation. This is a typical course for such a patient, an indolent, smoldering entity to an alarmingly overt one in just a few short hours. Surgical exploration of the abdomen with possible total colectomy or small bowel resection is life-saving in such cases.

Abdominal radiographs flat and upright (choice A) or chest films flat and upright (choice B) are useful for the diagnosis of free air and perhaps obstruction, but are not very useful in the diagnosis of early ischemic bowel. Once the bowel becomes necrotic with wall thickening, then radiographs may have more utility. Once this finding is present however, the patient will have deteriorated so severely that death would most certainly have occurred or will imminently occur.

A CT scan of the abdomen (choice C) is very helpful in these cases and is perfectly acceptable in cases where surgical exploration is not an option. The gold standard for diagnosis is direct visualization. The CT can offer radiographic suggestion of the disease but cannot, to the same degree as laparotomy, confirm the diagnosis. In fact, once a CT scan suggests it, patients are then scheduled for laparotomy for exploration.

An ultrasound of the abdomen (choice E) has no utility in these situations. This modality is very useful for biliary and pelvic diagnoses, but not colonic on bowel ones.

 

  A 52-year-old man is brought to the emergency department by his wife because he has had “bright red blood pouring from his mouth” for the past 20 minutes. The wife tells you that he has a 4-year history of alcoholic cirrhosis and he continues to drink 1 or 2 beers per day. He also has hypertension and hypercholesterolemia. Two days prior to admission, he had an episode of hematemesis and this morning, had an additional episode. He is diaphoretic with a blood pressure of 80/50 mm Hg and pulse of 110/min. Physical examination shows scleral icterus and mild jaundice, a tense abdomen, and cool, moist extremities. The most appropriate immediate action is to

  A. begin large volume intravenous fluids
  B. insert a Minnesota tube
  C. perform an emergent portal-systemic surgical shunt
  D. provide intravenous pressors for blood pressure control
  E. send a blood bank sample for type and crossmatching
Explanation:

The correct answer is A. The first priority in the management of any patient with GI bleeding is hemodynamic resuscitation and stabilization. IV fluids should be given immediately to maintain hemodynamic stability and adequate urinary output.

A Minnesota tube (choice B) is a type of orogastric tube with a gastric and esophageal balloon used for tamponade bleeding varices. They have fallen out of favor primarily due to the availability of endoscopes as well as the issue of pressure necrosis of the esophagus when they are utilized.

An emergent portal-systemic surgical shunt (choice C) would be a therapy of last resort if the bleeding cannot be controlled by less invasive measures.

Intravenous pressors for blood pressure control (choice D) is not indicated unless the patient proves refractory to therapy for his initial problem that is volume loss due to GI bleed. Therapy is volume resuscitation with blood and fluid.

A blood bank sample for type and crossmatching (choice E) is next on the priority list once the patient is hemodynamically stable.

 

A 52-year-old woman with hypertension comes to the office for a follow-up visit to discuss her recent diagnosis of colonic carcinoma. Her mother has breast and ovarian carcinomas, her grandmother had breast carcinoma, and her mother’s sister had stomach and ovarian carcinomas. Because of the significant family history, she underwent screening for colonic, ovarian, and breast carcinomas starting at an early age. In the last month, she noticed some change in her bowel habits and black colored stools. She was noted to have guaiac-positive stools and was referred for a colonoscopy. The colonoscopy examination confirmed a lesion in the right side of the colon, a biopsy of which identified it as an adenocarcinoma. No other lesions were noted in the rest of the colon on colonoscopic examination. She underwent an appendectomy at the age of 12 and a hysterectomy at the age of 31. Both surgical procedures were uneventful. The patient has 3 healthy children. At this time the most appropriate management is to advise the patient to consider

  A. biannual CT scans of the abdomen and pelvis
  B. gene therapy
  C. a right hemicolectomy
  D. a segmental colectomy
  E. a subtotal colectomy and bilateral salpingooophorectomy
Explanation:

The correct answer is E. This patient’s family history is strongly suggestive of hereditary non-polyposis colorectal carcinoma (HNPCC or Lynch syndrome). Lynch syndrome is due to mutations in mismatch repair genes, and are inherited as autosomal dominant. This predisposition runs in families to develop into endometrial, ovarian, breast, and gastric carcinomas. Affected individuals show predominance for right-sided cancers or multiple cancers and tend to be young when these cancers develop. They often develop metachronous colorectal cancer. Because of this predisposition, family members at risk should undergo biannual colonoscopy beginning at the age of 25, women should have an annual pelvic examinations with endometrial biopsies every 3 years and mammograms at an early age. In a woman who has completed child bearing, a total colectomy, and a hysterectomy, with a bilateral salpingooophorectomy should be considered.

A CT scan of the abdomen and pelvis (choice A) is indicated as a metastatic workup, but not as a therapy for right colonic carcinoma. Biannual CT scans are not a useful recommendation, as this patient is already diagnosed with colon cancer for which she requires therapy.

Gene therapy (choice B), when available, may prevent multiple carcinomas, but won’t treat an already established cancer.

Right hemicolectomy (choice C) in a patient with known Lynch syndrome may not prevent further cancers. Physicians should thoroughly discuss the possibilities of the development of multiple cancers and metachronous cancers in the future with these patients.

Segmental colonic resection (choice D) is not an option for the treatment of colonic carcinoma.

 

  A 61-year-old man with a history of ulcerative colitis comes to the clinic with a 1-week history of abdominal distension and occasional nausea. He has also had intermittent constipation and diarrhea for the past 3 weeks. Physical examination reveals an obese male with a distended abdomen with normal bowel sounds. The abdomen is diffusely tender to touch. There is no rebound or hepatosplenomegaly. Rectal examination shows heme-negative stool. His hematocrit is 44% and leukocyte count is 7000/mm3. The most appropriate next step in the management of this patient is to

  A. do a flexible sigmoidoscopy
  B. insert a rectal tube
  C. order an abdominal radiograph
  D. prepare him for a total colectomy
  E. send him for a colonoscopy
Explanation:

The correct answer is C. Patients with ulcerative colitis are at high risk for toxic megacolon, which is also associated with Clostridium difficile colitis. Toxic megacolon presents clinically as abdominal distension and bowel motility disturbances. The next step in evaluation is an abdominal radiograph which will demonstrate a distended large bowel. Toxic megacolon is the leading cause of death in patients with ulcerative colitis and carries a 40% mortality with each bout of toxic megacolon. Treatment consists of nasogastric tube insertion, no oral food or drink, rectal tube insertion, antibiotics, positional maneuvers, and ultimately surgery if there is no resolution in 2-5 days.

Flexible sigmoidoscopy (choice A) is a screening test used by some primary care physicians to evaluate for left-sided colon masses. The sigmoidoscope is unable to view the entire colon and specifically the right colon. Patients with ulcerative colitis are at high risk for toxic megacolon which is also found with Clostridium difficile colitis. Toxic megacolon presents clinically as abdominal distension and bowel motility disturbances. The next step in evaluation is the abdominal radiograph which will demonstrate a distended large bowel.

Rectal tube insertion (choice B) is a means to decompress a distended colon. A diagnosis must first be established by a plain abdominal radiograph.

Total colectomy (choice D) is necessary in some patients with ulcerative colitis because of multiple polyps or malignancies. Total colectomy is also necessary in patients with severe toxic megacolon, as a last resort treatment. Nevertheless an abdominal plain film is necessary first to evaluate if a distended colon is present.

Periodic colonoscopy (choice E) is necessary to assess for the development of colon cancer, which is much more common in patients with ulcerative colitis than in the population at large. Patients with ulcerative colitis are at a high risk for toxic megacolon which is also found with Clostridium difficile colitis. Toxic megacolon presents clinically as abdominal distension and bowel motility disturbances. The next step in evaluation is the abdominal radiograph which will demonstrate a distended large bowel.

  A 1-year-old boy is brought to the office by his mother because of a swelling in his left grointhat was initially noticed while giving him a bath 3 months ago. She feels that this swelling is completely asymptomatic and has grown minimally in size. The child was born without any difficulties, but developed a hydrocephalus, for which he underwent a ventricular peritoneal shunt. Since then he has had no other significant difficulties, besides some mild upper respiratory tract infections, which were well controlled. Physical examination reveals a left-sided easily reducible inguinal hernia and no other abnormalities. The most appropriate advice to this child’s mother is that

  A. a bilateral inguinal hernia repair is indicated
  B. a left inguinal hernia repair is indicated
  C. observation is all that is indicated
  D. repair should be delayed until the child is 2 years of age
  E. reversal of the ventricular peritoneal shunt is indicated
Explanation:

The correct answer is A. Inguinal hernias in children are usually indirect, resulting from failure of the obliteration of the processus vaginalis. Treatment requires high ligation and transection of the sac with or without excision of the distal component. Repair need not be delayed, unless the infant has associated medical problems. The complications from the hernia are most likely to occur during the first 6 months of life hence, a repair should be performed soon after the diagnosis. Contralateral exploration of the hernial sac should be performed routinely in the subset of children who are most likely to have a clinically occult hernia, children less than 2 years old, females less than 3 years old, patients with ventricular peritoneal shunts, and patients less than 2 years old with a left sided hernia.

Left sided repair of hernia (choice B) alone is not the correct option because in a child under the age of 2 with a ventricular peritoneal shunt, contralateral occurrence of the hernia is highly likely and should be repaired at the same time.

Observation (choice C) may lead to complications of hernia like irreducibility, incarcerations, strangulation and hence, the hernia should be repaired as soon as possible.

There is no indication to wait until the age of 2 years (choice D) to treat a hernia in a child.

Reversal of the ventricular peritoneal shunt (choice E) cannot be done as the patient has a hydrocephalus and this should not be considered as an option for a hernia in an infant.

 

 

A 56-year-old man is admitted to the hospital because of a 1-day history of acute, severe, cramping abdominal pain that radiated to his back. The pain was constant and exacerbated when he tried to eat some food. The patient attempted to self medicate with acetaminophen, but with no relief. The pain has slowly worsened and he has not had anything to eat or drink in over a day. On admission to the hospital, his serum amylase and lipase levels are elevated. The appropriate therapy is initiated and the patient has improvement in his pain. He is also started on a morphine patient-controlled anesthetic (PCA) with excellent results. Over the next 24 hours, he remains stable. A follow-up set of blood chemistries shows a BUN of 26 mg/dL and a creatinine of 1.0 mg/dL with an unchanged amylase and lipase. A right upper quadrant ultrasound shows gallstones with no ductal dilation. The patient’s other medications, besides the PCA, are diazepam for sleep and diphenhydramine. The most appropriate next step is to

  A. arrange for endoscopic retrograde cholangiopancreatography
  B. arrange for hepatobiliary iminodiacetic acid scan
  C. arrange for laparoscopic cholecystectomy
  D. arrange for an open cholecystectomy
  E. continue intravenous hydration and nil per os status
Explanation:

The correct answer is E. The correct therapy for pancreatitis is hydration, avoiding oral intake, and pain control. Given the success of this therapy for this patient over the previous 24 hours, it should continue.

Some centers will arrange for an ERCP (choice A) within 2 days of a pancreatitis episode to determine if papillotomy may be beneficial in abrogating the course of the disease. There is some clinical data for this but it is not yet common practice.

A HIDA scan (choice B) is used to diagnose cholecystitis, it is not indicated given that the abdominal ultrasound did not show evidence of this entity (gallbladder wall thickening, sludge).

Arranging for a laparoscopic cholecystectomy (choice C) or open cholecystectomy (choice D) will eventually be necessary given the pancreatitis and the presence of gallstones. However, this is not to be done presently and is generally done after the pancreatitis episode has passed (6-8 weeks).

 

A 54-year-old man with end-stage liver disease secondary to hepatitis C comes to the emergency department with fevers and mental status changes over the last 4 days. His wife reports that he has been compliant with his medications, which include furosemide, spironolactone, and lactulose up until today when he refused to take them. His temperature is 38.0 C (100.7 F), blood pressure is 100/70 mmHg, pulse is 103/min, and respirations are 19/min. Physical examination reveals a confused and slightly combative male with scleral icterus. His abdomen is distended with bulging flanks, shifting dullness, and a fluid wave. He has asterixis. There is no nuchal rigidity or photophobia. He is oriented to person but not place or time. The most appropriate next step in this patient’s management is to

  A. determine his ammonia level
  B. order a CT scan of the head
  C. perform a lumbar puncture
  D. perform paracentesis
  E. send a urine culture and sensitivity
Explanation:

The correct answer is D. This patient likely has spontaneous bacterial peritonitis. This diagnosis should be first on your list in any patient with ascites who presents with fevers, abdominal pain, change in mental status, or with other non-specific complaints. These patients need to have a paracentesis. This fluid is then sent to the lab for a cell count, culture, and Gram stain. The diagnosis of SBP can be made by seeing bacteria on a Gram stain, having more than 500 WBC or 250 PMNs in the cell count, or a positive peritoneal fluid culture. Patients with SBP need to be started on a third-generation cephalosporin.

Ammonia levels (choice A) can be elevated in patients that are encephalopathic. This patient is encephalopathic as evidenced by his asterixis and mental status changes. Ammonia levels can be followed if you want additional evidence that this patient’s medications are effectively causing a decrease in those levels, but ammonia should not be used to make the initial diagnosis of encephalopathy.

CT scan (choice B) is part of the work-up of mental status changes in the elderly, but acute stroke resulting in mental status changes in a 54-year-old would be less likely. CAT scan along with LP might be necessary if the paracentesis does suggest peritonitis.

Lumbar puncture (choice C) is also part of the work-up for mental status changes but this patient doesn’t have headache, high fevers, nuchal rigidity, or photophobia. Meningitis would be lower on the list of differential diagnoses in this patient and LP is not indicated as an initial diagnostic procedure.

Urine culture and sensitivity (choice E) is a test that we should all have a very low threshold to since urosepsis is a very common cause of fevers and mental status changes in the elderly and in the immunocompromised. This patient should have his urine evaluated, but the best answer is paracentesis since a positive urine culture would not rule out confounding peritonitis.

 

A 9-year-old girl is brought to the office by her mother because of “stomach aches” and constipation. She has been having one painful, hard bowel movement every 4 to 5 days. She admits that she never goes to the bathroom in school because she is too embarrassed, so she “holds it in until she is at home.” Many times she is so busy with after school activities such as ballet, piano, and gymnastics, that the “feeling” often passes by the time she gets home. The mother tells you that she complains of abdominal pain when this occurs, but it is too painful to defecate, so she continues to hold it in. She does not take any medications, has no medical illness, and has had normal bowel habits until 6 months ago. Physical examination shows mild abdominal tenderness, a hard mass in the lower abdomen, and a dilated rectum filled with a large amount of hard, guaiac negative, brown stool. The most appropriate next step is to

  A. obtain a consult with a pediatric gastroenterologist
  B. obtain an x-ray of the abdomen
  C. recommend a balanced diet containing whole grains, fruits, and vegetables
  D. recommend a phosphate soda enema
  E. schedule anal manometry
Explanation:

The correct answer is D. This patient has fecal impaction that is most likely caused by school bathroom avoidance. Fecal impaction can be diagnosed by finding a hard mass in the lower abdomen and a dilated rectum filled with a large amount of hard stool. This often occurs because of fecal stasis in the colon leading to an increased absorption of fluids, which in turn leads to the accumulation of large, hard stools that are painful to pass. This pain often leads to avoidance of defecation, even if the child is at home. The treatment includes disimpaction by oral or rectal medications, dietary and behavior modification. Disimpaction must be done before starting maintenance therapy.

A consult with a pediatric gastroenterologist (choice A) is usually only necessary after the child fails therapy, when an organic disease is suspected, or when there is complex management. This case seems like a simple case of school bathroom avoidance that can be treated with disimpaction, and dietary, behavior modification. If this fails, a consult may be considered.

An x-ray of the abdomen (choice B) is not necessary to establish the diagnosis of fecal impaction if the physical examination reveals a hard mass in the lower abdomen and a dilated rectum filled with a large amount of hard brown stool. It may be necessary, if the child refuses a rectal examination.

After a thorough history, physical examination, and disimpaction, you should recommend a balanced diet containing whole grains, fruits, and vegetables (choice C) and behavior modification to promote healthy bowel habits.

Anal manometry (choice E) is used in the evaluation of Hirschsprung disease, which is a cause of constipation in infants and rarely in school-age children. It is caused by a lack of colonic ganglion cells. Physical examination usually reveals a distended abdomen, a contracted anal sphincter, and a rectum devoid of stool. The physical exam of the child in this case is inconsistent with Hirschsprung disease. Also, she admits to school bathroom avoidance, which is probably the cause of her impaction.